daily q & a series

121
DAILY Q & A SERIES For Enrolment Join Telegram Channel: SIMPLIFIED_GS September 2021 Q1) World Water Development Report Released by a) UNESCO b) UNDP c) UNEP d) IUCN Ans : (a) India is majorly dependent on groundwater for irrigation and is pumping out the lion’s share of the global volume of groundwater. Around 70% of food production in India is done with the help of irrigation wells. o The UNESCO World Water Development Report states that India is the largest extractor of groundwater in the world. o In India, construction of irrigation wells does not require any clearance and no records are maintained of abandoned wells. Several hundred wells are constructed in India every day and even more are abandoned when they run dry. o The contribution of groundwater to national gross domestic product is never measured. o According to the Central Ground Water Board (CGWB, Ministry of Jal Shakti), with 230 billion metre cube of groundwater drawn out each year for irrigating agriculture lands in India, many parts of the country are experiencing rapid depletion of groundwater. The total estimated groundwater depletion in India is in the range of 122–199 billion metre cube Q2 ) Consider the correct statement about World Heritage Institute of Training and Research – Asia Pacific (WHITR-AP) : 1) a non-profit organization specialized in the area of heritage conservation.

Upload: others

Post on 19-Dec-2021

1 views

Category:

Documents


0 download

TRANSCRIPT

Page 1: DAILY Q & A SERIES

DAILY Q & A SERIES

For Enrolment Join Telegram Channel: SIMPLIFIED_GS

September 2021

Q1) World Water Development Report Released by a) UNESCO b) UNDP c) UNEP d) IUCN

Ans : (a)

India is majorly dependent on groundwater for irrigation and is pumping out the lion’s share of the global volume of groundwater. Around 70% of food production in India is done with the help of irrigation wells.

o The UNESCO World Water Development Report states that India is the largest extractor of groundwater in the world.

o In India, construction of irrigation wells does not require any clearance and no records are maintained of abandoned wells.

Several hundred wells are constructed in India every day and even more are abandoned when they run dry.

o The contribution of groundwater to national gross domestic product is never measured.

o According to the Central Ground Water Board (CGWB, Ministry of Jal Shakti), with 230 billion metre cube of groundwater drawn out each year for irrigating agriculture lands in India, many parts of the country are experiencing rapid depletion of groundwater.

The total estimated groundwater depletion in India is in the range of 122–199 billion metre cube

Q2 ) Consider the correct statement about World Heritage Institute of Training and Research – Asia

Pacific (WHITR-AP) :

1) a non-profit organization specialized in the area of heritage conservation.

Page 2: DAILY Q & A SERIES

DAILY Q & A SERIES

For Enrolment Join Telegram Channel: SIMPLIFIED_GS

2) an institute under the auspices of UNICEF.

3) an autonomous institution at the service of member states and associate

members of UNESCO.

a) 1 and 2

b) 2 and 3

c) 1 and 3

d) 1,2 and 3

Ans: c)

Recently, the World Heritage Institute of Training and Research – Asia Pacific (WHITR-AP) has recognised CEPT (Centre for Environmental Planning and Technology, Gujarat) University’s programme in conservation and regeneration as a commended case of Global Innovation on World Heritage Education.

o The WHITR-AP is a non-profit organization specialized in the area of heritage conservation.

o It is an institute under the auspices of UNESCO, it is the first one established in the developing countries.

o It is an autonomous institution at the service of member states and associate members of UNESCO.

Mission:

o To strengthen implementation of the World Heritage Convention 1972 in the Asia and the Pacific region.

World Heritage Convention:

o It is one of the most important global conservation instruments which was created in 1972.

Q3 ) Consider the INCORRECT statement about C40 Cities climate leadership group :

1) Mumbai joined C40 Cities group in December 2021.

2) There are 97 cities across the world connected in C40 Cities, who are pushed to develop and

implement climate action plans to address climate change by reducing greenhouse gas

emission significantly.

a) 1 only

b) 2 only

c) Both 1 and 2

d) Neither 1 nor 2

Ans : a)

The Brihanmumbai Municipal Corporation (BMC) is drafting a Mumbai Climate Action Plan (MCAP) in a bid to tackle climate challenges. It recently launched a website for the same.

Page 3: DAILY Q & A SERIES

DAILY Q & A SERIES

For Enrolment Join Telegram Channel: SIMPLIFIED_GS

This Plan will look at climate resilience with mitigation and adaptation strategies by focusing on six areas — sustainable waste management, urban greening and biodiversity, urban flooding and water resource management, building energy efficiency, air quality, and sustainable mobility.

The plan is expected to be ready by November ahead of the United Nations Climate Change (COP26) conference. Why does Mumbai need a climate action plan?

As per a study conducted by the World Resource Institute (WRI) India on Mumbai’s vulnerability assessment, the city will face two major climate challenges — the rise in temperature, and extreme rain events which will lead to flooding.

A recent report from the Intergovernmental Panel on Climate Change (IPCC) has warned that at least 12 Indian coastal cities including Mumbai will face sea rise of 0.1 metres to 0.3 metres in next three decades due to climate change. C40 Cities climate leadership group

This has been done following Mumbai’s commitment to C40 Cities climate leadership group. Mumbai joined C40 Cities group in December 2020. There are 97 cities across the world connected in C40 Cities, who are pushed to develop and implement

climate action plans to address climate change by reducing greenhouse gas emission significantly.

Q4) Nyongbyon Nuclear Scientific Research Center is located in

a) North Korea b) China c) South Korea d) Iran

Ans : a)

The United Nations’ nuclear watchdog described the resumption of operations over the weekend at North Korea’s Yongbyon nuclear reactor as “deeply troubling.”

The International Atomic Energy Agency (IAEA) stated that while monitors have not been granted access to the Yongbyon site, there are “indications” that the five-megawatt reactor is once more producing plutonium for the first time since December 2018.

The Nyongbyon Nuclear Scientific Research Center is North Korea's major nuclear facility, operating its first nuclear reactors.

It is located in Nyongbyon County in North Pyongan Province, about 100 km north of Pyongyang.

Q5) With reference to UNSC Resolution 2593, adopted recently, consider the following statements:

1. The resolution called upon host States to promote accountability for the killing and all acts of violence against the UN peacekeeping personnel.

2. It specifically mentions individuals designated by the UNSC resolution 1267, which includes the Lashkar-e-Taiba and the Jaish-e-Mohammad.

3. All the 15 members of the Council voted unanimously in favour of the resolution.

Which of the statement(s) given above is/are correct?

Page 4: DAILY Q & A SERIES

DAILY Q & A SERIES

For Enrolment Join Telegram Channel: SIMPLIFIED_GS

a) 2 only

b) 1 and 3 only

c) 2 and 3 only

d) 1, 2 and 3

Ans: a)

UNSC Resolution 2593

The resolution demanded that Afghan territory not be used to threaten or attack any country and reiterated the importance of combating terrorism in Afghanistan.

The resolution adopted by a vote of 13 in favour with two abstentions (Russian Federation and China).

The resolution also urged Talibans’ to assist the safe evacuations of all Afghan nationals wishing to leave the country.

It specifically mentions individuals designated by the UNSC resolution 1267, which includes the Lashkar-e-Taiba (LeT) and the Jaish-e-Mohammad (JeM).

Q6) Consider the following statements:

1. The river flows in both India and Bangladesh and act as a natural boundary between Jaintia and Khasi Hills.

2. Over this river hangs a single span suspension bridge called Dawki Bridge. 3. The river is considered as the cleanest river in India and in some parts is as transparent as

crystal and you can actually see the river bed. 4. The river passes through the Asia’s Cleanest Village, Mawlynnong.

Identify the river that correctly matches with the above description:

a) Kynshi River

b) Umngot River

c) Jinjiram River

d) Simsang Ri

Ans : b)

The Meghalaya government has recently scrapped an agreement with private power developers to execute the proposed 210 MW Umngot hydroelectric project following protests.

Page 5: DAILY Q & A SERIES

DAILY Q & A SERIES

For Enrolment Join Telegram Channel: SIMPLIFIED_GS

Umngot River

It flows both in India & Bangladesh. It separates East Khasi Hills District from Jaintia Hills District by creating a boundary in between

the two. It is the natural boundary between Ri Pnar (of Jaintia Hills) with Hima Khyrim (of Khasi Hills). Over the river hangs a single span suspension bridge called Dawki Bridge. Umngot River is considered as the cleanest river in India and in some parts is as transparent as

crystal and you can actually see the river bed. Umngot River is also called by the name Dawki River, which has greenish-bluish color

transparent water. The Umngot River passes through the village of Mawlynnong in Meghalaya, close to India’s

border with Bangladesh, which is touted as “Asia’s Cleanest Village"

Q7) Consider the following statements with respect to Self-Reliant India (SRI) Fund

1. It is an Alternative Investment Fund to provide growth capital for Micro, Small and Medium Enterprises (MSMEs).

2. The fund will have a Mother Fund/Daughter Fund structure with a corpus of Rs.10,000 crore as Mother Fund.

3. Govt. of India will be the sole anchor investor of the fund.

Which of the statement(s) given above is/are correct?

a) 1 and 2 only

b) 2 and 3 only

c) 2 and 3 only

d) 1, 2 and 3

Ans : d)

It is an Alternative Investment Fund (AIF), for providing growth capital to the Micro, Small and Medium Enterprises (MSMEs), through equity/quasi-equity/equity like structured instruments.

This will facilitate them to move towards listing on the Stock Exchanges and in becoming National and International Champions by growing beyond the bracket of MSME.

The Fund is to be anchored by NSIC Venture Capital Fund Limited (NVCFL). NVCFL is a wholly owned subsidiary of The National Small Industries Corporation. Structure of the fund - SRI Fund will have a Mother Fund / Daughter Fund structure with a

corpus of INR 10,000 crore as Mother Fund. Govt. of India will be the sole anchor investor and provide an initial budgetary support of Rs.

10,000 crore to the Mother Fund in phased manner. Mother Fund will provide funds only to the Daughter Funds for onward investment as growth

capital, while the investment in MSMEs will be done by the Daughter Funds.

Page 6: DAILY Q & A SERIES

DAILY Q & A SERIES

For Enrolment Join Telegram Channel: SIMPLIFIED_GS

Both the Mother and Daughter Funds will be duly registered as Alternate Investment Funds with SEBI.

Fund Life - The SRI Fund will have a life of 15 years and it will be a revolving fund.

Q8) Which of the following are the impacts of using Antibiotics in Soil?

1. Decrease in ability of the Soil to store carbon 2. Change in Soil’s bacterial community structure 3. More carbon in the atmosphere in the form of CO2

Choose the correct option:

a) 2 only

b) 1 and 3 only

c) 2 and 3 only

d) 1, 2 and 3

Ans : d)

Effects of Antibiotics in soil

Carbon in soils exposed to manure from cows administered with antibiotics, travelled into the above-ground plant material, to the roots of the plants, into the soil and respired back out as CO2 much faster than any of the others.

This means that when antibiotics are used, less carbon is stored in the soil and more is lost to the atmosphere as CO2

It can change soil microbiome and decrease their ability to store carbon It also affects soil respiration and elemental cycling It also caused changes in the soil's fungal and bacterial ratios as well as altered the bacterial

community structure.

Q9) Which of the following countries has recently declared Food Emergency due to drain of Forex to finance imports?

a) Morocco

b) Sri Lanka

c) Afghanistan

d) Sudan

Page 7: DAILY Q & A SERIES

DAILY Q & A SERIES

For Enrolment Join Telegram Channel: SIMPLIFIED_GS

Ans : b)

Sri Lanka has recently declared a state of emergency over food shortages as private banks run out of foreign exchange to finance imports.

The emergency regulation allows government officials to seize food stocks held by traders and arrest people who hoard essential food.

Sri Lanka’s foreign reserves fell to $2.8 billion at the end of July 2021, from $7.5 billion in November 2019.

Tourism was the only vital source of foreign exchange earnings, but that too suffered due to the coronavirus pandemic.

It has more foreign debt payments of $1.5 billion each due in the next 12 months. It has already paid $1.3 billion so far.

Q10) Consider the following statements about Cyanobacteria

1. Cyanobacteria are prokaryotes belong to Monera kingdom and often called as Blue-Green Algae.

2. They are mainly aquatic that phosynthesize their own food. 3. It is capable of producing toxins that can be ingested by livestocks and enter the food chain.

Which of the statements given above is/are correct?

a) 1 only

b) 2 only

c) 2 and 3 only

d) 1, 2 and 3

Ans: d)

Cyanobacteria

Scientists have identified a set of genes in cyanobacteria that can help boost energy yield from photosynthesis, and also pave way for a sustainable biotech production.

Cyanobacteria are aquatic and photosynthetic and are often called blue-green algae. They have the distinction of being the oldest known fossils, more than 3.5 billion years old. The cyanobacteria have also been tremendously important in shaping the course of evolution

and ecological change throughout earth's history. The oxygen atmosphere that we depend on was generated by numerous cyanobacteria during

the Archaean and Proterozoic Eras. The other great contribution of the cyanobacteria is the origin of plants. The chloroplast with

which plants make food for themselves is actually a cyanobacterium living within the plant's cells.

Page 8: DAILY Q & A SERIES

DAILY Q & A SERIES

For Enrolment Join Telegram Channel: SIMPLIFIED_GS

Dead and dying cyanobacteria, which contain preformed toxins saccumulate on the surface of bodies of water and are ingested by livestock.

The bacterial species share with plants an enzyme called rubisco, which plays a key role in photosynthesis.

Q11) ‘World Social Protection Report 2020–22 released by a) UNDP b) International Labour Organization c) WTO d) World Economic Forum

Ans : (b)

Recently, a International Labour Organization report titled ‘World Social Protection Report 2020–22’ has revealed that, globally 4.1 billion people are living without any social safety net of any kind.

The report highlighted that the pandemic response was uneven and insufficient. Thereby, Covid-19 has further underscored the critical importance of achieving universal social protection.

ILO is a specialized agency of the United Nations. It is the only tripartite UN agency. It brings together governments, employers and workers of 187 member States, to set labour standards, develop policies and devise programmes promoting decent work for all women and men.

Q12 ) Maghreb region, Recently seen in the news, located in

a) Algeria

b) UAE

c) USA

d) UK

Ans: a)

Recently, the Indian and Algerian navies participated in a maiden naval exercise off the Algerian coast in a bid to increase maritime cooperation.

The naval exercise with Algeria is crucial for India as it is strategically located in the Maghreb region (region of North Africa bordering the Mediterranean Sea) and is the largest country of Africa.

Q13 ) With reference to the judges of the Supreme Court, consider the following statements:

1. With the swearing-in of the nine new judges, the strength of the Supreme Court has risen to 33,

including the CJI, out of the sanctioned strength of 34.

2. With the new appointments, the number of women judges in the Supreme Court has gone up to

four from the existing one, Justice Indira Banerjee.

Page 9: DAILY Q & A SERIES

DAILY Q & A SERIES

For Enrolment Join Telegram Channel: SIMPLIFIED_GS

Which of the statements given above is/are correct?

a. 1 only

b. 2 only

c. Both 1 and 2

d. Neither 1 nor 2

Ans : c)

Chief Justice of India (CJI) N.V. Ramana read out the oath of allegiance to the Constitution to nine new judges of the Supreme Court.

With the swearing-in of the nine new judges, the strength of the Supreme Court has risen to 33, including the CJI, out of the sanctioned strength of 34.

The occasion was historic on several fronts. o It was the first time as many as nine judges were sworn in to the Supreme Court in a single stroke. o Three of them are women judges. One of them, Justice B.V. Nagarathna, is slated to be CJI in 2027. o With the new appointments, the number of women judges in the Supreme Court has gone up to four

from the existing one, Justice Indira Banerjee. This is the highest number of women judges ever in the Supreme Court.

o It was also the first time the court allowed a live telecast of the ceremony. The apex court Collegium had on August 17 recommended these nine names for appointment as judges

of the top court. Later, the President had signed the warrants of their appointment as apex court judges

Q14) The South China Sea, which lies between which of the following countries?

1. China

2. Taiwan

3. Philippines

Select the correct answer using the code given below:

a. 1 only

b. 1 and 2 only

Page 10: DAILY Q & A SERIES

DAILY Q & A SERIES

For Enrolment Join Telegram Channel: SIMPLIFIED_GS

c. 2 and 3 only

d. 1, 2 and 3

Ans : d)

From September 1, 2021, China’s new maritime rules designed to control the entry of foreign vessels in what Beijing calls “Chinese territorial waters” take effect.

Foreign vessels, both military and commercial, will be henceforth required to submit to Chinese supervision in “Chinese territorial waters,” as per the new law.

The move is expected to have far-reaching consequences for passage of vessels, both commercial and military, in the disputed South China Sea, East China Sea and Taiwan Strait, and is likely to escalate the existing tension with the US and its neighbours in the region. South China Sea

The South China Sea, which lies between China, Taiwan, the Philippines, Brunei, Malaysia, Indonesia and Vietnam, is of great economic importance globally.

The waters around China are hotly contested. Under a “nine-dash line” map, China claims most of the South China Sea as its sovereign territory. This claim is contested by its neighbours in the region and by the United States, which, though it has no claim in the Sea, backs the smaller nations in the fight against Chinese overreach.

Currently, international maritime activities are governed by an international agreement called the United Nations Convention on the Law of the Sea (UNCLOS) of which China, India and over a hundred other countries are signatories (the US, significantly, is not).

Accordingly, states have the right to implement territorial rights up to 12 nautical miles into the sea. The UNCLOS also states that all vessels have the right of “innocent passage” through this region – China’s new law violates this.

Q15) Consider the Incorrect statement about Deepor Beel :

1) Deepor Beel is a Ramsar site

2) it is a part of wildlife sanctuary (Rani Reserve forest) in Guwahati, Assam.

3) The site is an important destination for many migratory species of birds

a) 1 and 2

b) 2 and 3

c) 1 and 3

d) None of these

Ans: d)

Degradation of Assam’s Deepor Beel due to anthropogenic forces

More on this:

Page 11: DAILY Q & A SERIES

DAILY Q & A SERIES

For Enrolment Join Telegram Channel: SIMPLIFIED_GS

Human activities which have caused this destruction– dumping of garbage, construction of roads and railways, construction of warehouses, tourism activities

This degradation has caused death of elephants in railway accidents, fewer elephants visiting the wetland due to deteriorating quality of water due garbage dumping, reduction in the number of fishes available for fishermen etc

1. Deepor Beel is a Ramsar site and a part of it is also wildlife sanctuary (Rani Reserve forest) in Guwahati, Assam.

2. Deepor Beel is a permanent freshwater lake, in an earlier channel of the Brahmaputra River, to the south of the main river.

3. The site is an important destination for many migratory species of birds

Q16) Consider the following statements with respect to ZAPAD 2021

1. It is one of the theatre level exercises of Russian armed forces that focus primarily on operations against terrorists.

2. India is one among the 9 participating countries that are invited for the exercise. 3. China, Pakistan is taking part as observers in this multinational exercise.

Which of the statement(s) given above is/are correct?

a. 1 and 2 only

b. 1 and 3 only

c. 2 and 3 only

d. 1, 2 and 3

Ans : d)

A contingent of 200 Army personnel of Indian Army will participate in the multinational Exercise ZAPAD 2021 being held at Nizhniy, Russia.

ZAPAD 2021

It is one of the theatre level exercises of Russian armed forces that focus primarily on operations against terrorists.

Of the 17 countries invited for the exercise, there are nine participating countries and eight Observers.

The nine Participating countries includes Mongolia, Armenia, Kazakhstan, Tajikistan, Kyrgyzstan, Serbia, Russia, India and Belarus.

The other eight Observer countries are Pakistan, China, Vietnam, Malaysia, Bangladesh, Myanmar, Uzbekistan and Sri Lanka.

Page 12: DAILY Q & A SERIES

DAILY Q & A SERIES

For Enrolment Join Telegram Channel: SIMPLIFIED_GS

Q17) Panjshir Valley, often seen in the news recently, is located in?

a. Pakistan

b. Mongolia

c. Tajikistan

d. Afghanistan

Ans : d)

The Taliban had recently called on fighters in the Panjshir Valley to lay down their arms. The Panjshir Valley is the centre of Afghanistan’s most important pocket of armed anti-Taliban

forces. The valley stood strong against the Taliban rule from 1996-2001 before the US-led foreign

troops invaded Afghanistan.

Panjshir Valley

Located 150 km north of Kabul, Afghanistan, the Valley is near the Hindu Kush mountain range. It’s divided by the Panjshir River and ringed by the Panjshir Mountains in the north and the

Kuhestan mountains in the south. The mountain tops are covered by snow throughout the year. This difficult terrain makes the Valley a nightmare for invaders.

Q18) Consider the following statements about Direct Seeding of Rice

1. It involves no nursery preparation of seeds and requires less water than conventional transplanting method.

2. It uses chemical herbicides. 3. The seed requirement is very high.

Which of the statements given above is/are correct?

a. 1 only

b. 1 and 3 only

c. 2 and 3 only

d. 1, 2 and 3

Page 13: DAILY Q & A SERIES

DAILY Q & A SERIES

For Enrolment Join Telegram Channel: SIMPLIFIED_GS

Ans : d)

In the conventional transplantation, farmers prepare nursery seedbed to be transplanted but in DSR, the paddy seeds are directly drilled into the field by a tractor-powered machine.

Conventional transplantation required the paddy field to be flooded to prevent the growth of herbicides.

In DSR, chemical herbicides are used in two stages – Pre-emergent and post-emergent and it leads to water savings.

Labour - About three labourers are required to transplant one acre of paddy in a single day. The seed requirement for DSR is also higher, at 8-10 kg/acre, compared to 4-5 kg in

transplanting. Laser land levelling, which costs Rs 1,000/acre, is compulsory in DSR. This is not so in

transplanting

Q19) Consider the following statements about new variant of SARS-COV

1. The latest variant B.1.621 has been given label ‘Mu’ and classified as ‘Variant of Interest’. 2. It was first identified in Colombia in 2021.

Which of the statements given above is/are correct?

a. 1 only

b. 2 only

c. Both 1 and 2

d. Neither 1 nor 2

Ans : c)

‘Mu’ Variant

The latest variant of interest, B.1.621, has been given the WHO label ‘Mu’ and classified as ‘variant of interest’.

This includes the descendent Pango lineage B.1.621.1. It was first identified in Colombia in January 2021. There have been sporadic reports of the variant from South America and Europe. The Mu variant has a constellation of mutations that indicate potential properties of immune

escape.

Q20) Consider the following statements about Allocation of Special Drawing Rights (SDR) to India

1. India has recently been allocated 25% of overall SDRs general allocation made to the IMF’s member countries.

Page 14: DAILY Q & A SERIES

DAILY Q & A SERIES

For Enrolment Join Telegram Channel: SIMPLIFIED_GS

2. SDR allocation is made to IMF countries to reduce their reliance on more expensive domestic or external debt for building reserves.

Which of the statements given above is/are not correct?

a. 1 only

b. 2 only

c. Both 1 and 2

d. Neither 1 nor 2

Ans: a)

Special Drawing Rights (SDR) by IMF

The SDR is based on a basket of international currencies comprising the US dollar, Japanese yen, euro, pound sterling and Chinese Renminbi.

It is not a currency, nor a claim on the IMF, but is potentially a claim on freely usable currencies of IMF members.

IMF has made an allocation of SDR 12.57 billion (equivalent to around $17.86 billion at the latest exchange rate) to India.

It is about 2.75% of overall SDRs allocated to member countries. IMF makes the general SDR allocation to its members in proportion to their existing quotas in

the IMF. It is one of the components of the foreign exchange reserves of a country.

Q21) ATL Space Challenge 2021 is an Initiative of 1) Atal innovation Mission 2) ISRO 3) CBSE 4) Niti Aayog

a) 1 only b) 2 and 3 c) 1,3 and 4 d) 1,2,3 and 4

Ans : (d)

Atal Innovation Mission (AIM), NITI Aayog in collaboration with the Indian Space Research Organisation (ISRO) and Central Board of Secondary Education (CBSE) successfully launched the ATL Space Challenge 2021 for all school students across the country.

The challenge has been designed for all the school students, mentors and teachers across the country who not only are associated with schools having ATL labs but for all the non ATL schools as well.

Page 15: DAILY Q & A SERIES

DAILY Q & A SERIES

For Enrolment Join Telegram Channel: SIMPLIFIED_GS

This is to ensure that students of classes 6 to 12 are given an open platform where they can innovate and enable themselves to solve digital age space technology problems.

The ATL Space Challenge 2021 aligns with the World Space Week 2021 which is being observed from 4 to 10 October each year at the global level in order to celebrate the contributions of space science and technology.

Students can create a solution that can be implemented and adopted leveraging technologies such as: o Explore Space o Reach Space o Inhabit Space o Leverage Space

Q22 ) Saṃvatsari is related to which of the following religion?

a) Buddhism

b) Jainism

c) Hinduism

d) Sikhism

Ans: b)

The Prime Minister Narendra Modi has greeted the people on the occasion of Samvatsari.

Saṃvatsarī is the last day of Paryushana Shwetambar sect of Jainism. It falls on Shukla Panchami each year in the Jain calendar month of Bhadrapada, somewhere between

the middle of August and September in the Gregorian calendar. On this day, Jains forgive and seek forgiveness for their mistakes committed knowingly or unknowingly

from all the living beings. A yearly, elaborate penitential retreat called "samvatsari pratikramana" is performed on this day. After the pratikramana, Jains seek forgiveness from all the creatures of the world, including friends and

relatives by uttering the phrase — Micchami Dukkadam or its variants like "Khamau Sa", "Uttam Kshama" or "Khamat Khamna". Q23 ) FOSS4Gov Innovation Challenge is an initiative of:

a) Niti Aayog

b) Union Ministry of Finance

c) Union Ministry of Electronics and IT

d) World Bank

Ans : c)

The government has extended the last date of participation and submission in FOSS4Gov Innovation Challenge till 15th of September.

Ministry of Electronics and IT has appealed to the Free and Open Source Software (FOSS) innovators to participate in large numbers to drive the FOSS revolution to its maximum potential.

Page 16: DAILY Q & A SERIES

DAILY Q & A SERIES

For Enrolment Join Telegram Channel: SIMPLIFIED_GS

In July this year, on the occasion of 7-year anniversary of MyGov, the Ministry had launched #FOSS4Gov Innovation Challenge to accelerate adoption of Free and Open Source Software in Government and build Indian FOSS Ecosystem.

Q24) Recently, the first Indigenously Designed High Ash Coal Gasification Based Methanol Production Plant has been opened in

a) Delhi b) Mumbai c) Lucknow d) Hyderabad

Ans : d)

Recently, the first Indigenously Designed High Ash Coal Gasification Based Methanol Production Plant has been opened in Hyderabad.

With this, Government owned engineering firm BHEL (Bharat Heavy Electricals Limited) has successfully demonstrated a facility to create methanol from high ash Indian coal.

o Methanol is utilized as a motor fuel, to power ship engines, and to generate clean power all over the world. However, the majority of worldwide methanol production is derived from natural gas, which is a relatively easy process.

o Since India doesn’t have much of the natural gas reserves, producing methanol from imported natural gas leads to outflow of foreign exchange and is uneconomical because of higher prices.

o The next best option is to utilise India's abundant coal. However, due to the high ash percentage of Indian coal, most internationally accessible technology will not be adequate.

o To address this issue, BHEL successfully demonstrated a facility to create 0.25 TPD (Ton per Day) Methanol from high ash Indian coal using a 1.2 TPD Fluidized bed gasifier.

Q25) Consider the Incorrect statement about Thamirabarani civilisation :

1) Thamirabarani civilisation is at least 3,200 years old.

2) Thamirabarani civilisation is related to Tamil Nadu.

a) 1 only

b) 2 only

c) Both 1 and 2

d) Neither 1 nor 2

Ans: d)

The Thamirabarani civilisation in Tamil Nadu is at least 3,200 years old, reveals carbon dating done on organic material retrieved from archeological excavations in Sivakalai, Thoothukudi district. It could lead to evidence that there was a city civilisation (Porunai River (Thamirabarani) civilization) in south India as long back as 3,200 years ago, the later part of the Indus Valley Civilisation.

Q26) Recently, in a major discovery, footprints of three species of dinosaurs have been found in

Page 17: DAILY Q & A SERIES

DAILY Q & A SERIES

For Enrolment Join Telegram Channel: SIMPLIFIED_GS

a) Thar Desert

b) Sahara Desert

c) Gobi Desert

d) None of these

Ans : a)

Recently, in a major discovery, footprints of three species of dinosaurs have been found in the Thar desert in Rajasthan’s Jaisalmer district.

It proves the presence of the giant reptiles in the western part of the State.

The footprints belong to three species of dinosaurs - Eubrontes cf. giganteus, Eubrontes glenrosensis and Grallator tenuis.

The footprints were 200 million years old.

The dinosaur species are considered to be of the theropod type, with the distinguishing features of hollow bones and feet with three digits (like fingers).

Theropod is any member of the dinosaur subgroup Theropoda, which includes all the flesh-eating dinosaurs.

All the three species, belonging to the early Jurassic period, were carnivorous.

The 'Age of Dinosaurs' (the Mesozoic Era - 252-66 Million Years Ago - MYA) included three consecutive geologic time periods (the Triassic, Jurassic, and Cretaceous Periods). Different dinosaur species lived during each of these three periods.

Q27) World Economic Situation and Prospects Report released by

a) World Bank

b) IMF

c) United Nations Department of Economic and Social Affairs

d) World Economic Forum

Ans : c)

The United Nations Conference on Trade and Development (UNCTAD), United Nations Department of Economic and Social Affairs (UN DESA) and the five United Nations Regional Economic Commissions have jointly released the World Economic Situation and Prospects.

Q28) Who became the first IAS officer to win a Paralympic medal?

a) Sumit Antil

b) Suhas Yathiraj

c) Devendra Jhajharia

Page 18: DAILY Q & A SERIES

DAILY Q & A SERIES

For Enrolment Join Telegram Channel: SIMPLIFIED_GS

d) Krishna Nagar

Ans : b)

Noida District Magistrate Suhas Suhas Yathiraj became the first IAS officer to win a medal in Badminton at the Tokyo 2020 Paralympics Games. The 38-year-old made history by winning a silver medal in the men’s singles SL4 category.

Q29) Which Indian spacecraft has completed 9000 orbits around the Moon?

a) Chandrayaan-2

b) Chandrayaan-1

c) Mangalyaan

d) None of the Above

Ans : a)

The Indian Space Research Organisation (ISRO) informed on September 6, 2021 that the Chandrayaan-2

spacecraft has completed 9,000 orbits around the moon. The spacecraft has detected minor elements

of manganese and chromium at few places on the lunar surface through remote sensing during the

intense solar flare events.

Q30) World's first all-civilian space mission-Inspiration4 be launched by

a) ISRO

b) SpaceX

c) China

d) Japan

Ans: b)

The world’s first all-civilian mission to orbit will be launched on September 15, 2021. The mission,

named Inspiration4 will be launched aboard Falcon 9 rocket of SpaceX from NASA's Kennedy Space

Centre in Florida, US. The mission will have no professional astronauts, as all four mission members are

private citizens.

Q31) Consider the Incorrect statement about US-India Agenda 2030 Partnership : 1) aim is to create stronger bilateral cooperation on actions in the current decade to meet the

goals of the Paris Agreement. 2) India elevated the India-US energy dialogue to a strategic energy partnership in 2018.

a) 1 only b) 2 only c) Both 1 and 2

Page 19: DAILY Q & A SERIES

DAILY Q & A SERIES

For Enrolment Join Telegram Channel: SIMPLIFIED_GS

d) Neither 1 nor 2

Ans : (d)

Recently, the revamped US-India Strategic Clean Energy Partnership SCEP was launched during the ministerial meeting of the Petroleum and Natural Gas Ministry with the US Ministry of Energy.

The SCEP was launched in accordance with the US - India Climate and Clean Energy Agenda 2030 Partnership announced by both countries at the Leaders’ Summit on Climate held earlier this year (2021).

US-India Agenda 2030 Partnership:

o The aim is to create stronger bilateral cooperation on actions in the current decade to meet the goals of the Paris Agreement.

o The Partnership will proceed along two main tracks: the Strategic Clean Energy Partnership and the Climate Action and Finance Mobilization Dialogue.

o India elevated the India-US energy dialogue to a strategic energy partnership in 2018.

Q32 ) Recently , pgSIT seen in the news, is associated with

a) an analysis showed a changing rainfall pattern in North East (NE) India

b) It is a new scalable genetic control system that uses a CRISPR-based

approach to engineer deployable mosquitoes that can suppress populations.

c) Association for cooperative movement

d) None of these

Ans: b)

Recently, researchers have created a system that restrains populations of mosquitoes by leveraging advancements in Clustered Regularly Interspaced Short Palindromic Repeats (CRISPR)-based genetic engineering.

Mosquitoes infect millions each year with debilitating diseases such as dengue and malaria.

pgSIT: o It is a new scalable genetic control system that uses a CRISPR-based approach to engineer

deployable mosquitoes that can suppress populations.

Males don't transmit diseases so the idea is to release more and more sterile males.

The population of mosquitos can be suppressed without relying on harmful chemicals and insecticides.

Page 20: DAILY Q & A SERIES

DAILY Q & A SERIES

For Enrolment Join Telegram Channel: SIMPLIFIED_GS

Q33 ) Which of the following country is considering proposals of slashing livestock numbers by 30 per

cent by forcing farmers to sell their emission rights and even their land to the state?

a) Netherlands

b) France

c) Germany

d) U.K

Ans : a)

The Netherlands is considering proposals, which are the most radical of its kind in Europe, of slashing livestock numbers by 30 per cent by forcing farmers to sell their emission rights and even their land to the state.

This comes following growing public debate in the country over the effects of livestock production on human health and the environment. The debate had found a renewed impetus after the Q fever epidemic which hit the most densely populated livestock areas in the country in 2007-10.

The Netherlands is the EU’s biggest meat exporter. It has also one of the largest livestock industries in Europe, with more than 100 million cattle, chickens and pigs. What is the reason behind such a plan?

The Netherlands is battling an acute climate crisis caused by an excess of nitrogen emissions. The concern regarding livestock stems from the fact that they produce manure which, when mixed with urine, releases ammonia, which is a nitrogen compound.

This ammonia, via farm runoff, can get into waterbodies, in which case the excessive nitrogen will damage sensitive natural habitats. Nitrogen can lead to algae that deplete oxygen at the surface of the water.

A study published in Elsevier last year stated that livestock production leads to alarming nitrate pollution of groundwater.

Q34) India's largest open air fernery was recently inaugurated in:

a) Ranikhet, Uttarakhand

b) Kochi, Kerala

c) Jaipur, Rajasthan

Page 21: DAILY Q & A SERIES

DAILY Q & A SERIES

For Enrolment Join Telegram Channel: SIMPLIFIED_GS

d) Surat, Gujarat

Ans : a)

India's largest open air fernery was inaugurated in Uttarakhand’s Ranikhet.

The fernery is home to a large number of fern species, some of which are endemic to the state, some hold medicinal value while some are threatened species that demand care and conservation.

The newly inaugurated fernery is one of the biggest ferneries in India. The fernery has the largest collection of fern species, second to only Jawaharlal Nehru Tropical Botanical Garden and Research Institute (TBGRI), Thiruvananthapuram.

However, it is the country’s first open-air fernery in natural surroundings which is not under any poly-house/ shade house.

It has been developed by the Research Wing of Uttarakhand Forest Department over a period of three years, under Central Government’s CAMPA scheme.

The CAMPA or Compensatory Afforestation Management Funds Management and Planning Authority (CAMPA) by the Ministry of Environment and Forests (MoEF) was introduced in 2004 to accelerate activities for the preservation of natural forests, management of wildlife, infrastructure development in forests, and other allied works.

Q35) The National Company Law Tribunal formed on the recommendations of ?

a) Rajshekhar Committee

b) Eradi Committee

c) Malhotra committee

d) Narshimahn committee

Ans: b)

The government has appointed 31 people as judicial, technical and accountant members at the National Company Law Tribunal (NCLT) and the Income Tax Appellate Tribunal (ITAT).

These developments assume significance as they come amid the Supreme Court flagging concerns about vacancies in various tribunals.

About NCLT:

It is a quasi-judicial body in India that adjudicates issues relating to companies in India. Established on 1st June, 2016 (Companies Act, 2013). Formed based on the recommendations of the Justice Eradi Committee. It deals with matters mainly related to companies law and the insolvency law. Term of members: Appointments will be for five years from the date of assumption of

charge or till attaining the age of 65 or until further orders.

Q36) Consider the correct statement about Subansiri River :

1) Subansiri River (gold river), originates in the Tibet Plateau and enters India through Miri hills in Arunachal Pradesh.

Page 22: DAILY Q & A SERIES

DAILY Q & A SERIES

For Enrolment Join Telegram Channel: SIMPLIFIED_GS

2) It is the largest tributary of Brahmaputra River.

a) 1 and 2

b) 2 only

c) 1 only

d) Neither 1 nor 2

Ans : a)

Subansiri Lower Hydroelectric Project (SLHEP), is an under-construction gravity dam on the Subansiri river along the border of Assam and Arunachal Pradesh.

Subansiri River (gold river), originates in the Tibet Plateau and enters India through Miri hills in Arunachal Pradesh.

It is the largest tributary of Brahmaputra River. The project is being developed by the state-run National Hydro Power Corporation

(NHPC). It will be the single largest hydroelectric plant in India when completed. The project is

expected to be completed in 2023.

Q37) Which state has become the second state in India to complete 100% covid vaccination of the first dose for all its eligible residents?

a) Goa

b) Himachal Pradesh

c) Odisha

d) Mizoram

Ans : a)

Goa has become the second state in India to complete 100% covid vaccination of the first dose for all its eligible residents.

Himachal Pradesh was the first state to achieve this feat.

Q38) Who Published the sensational “Sudesa Geethangal” in 1908.

a) Dada bhai nauroji

b) Ajit Singh

c) Mahakavi Subramania Bharati

d) Arvindo Ghosh

Ans : c)

Recently, the Vice-President paid tributes to Mahakavi Subramania Bharati on his 100th death centenary.

Brief Profile: Indian writer of the nationalist period (1885-1920) who is regarded as the father of the modern Tamil style.

Page 23: DAILY Q & A SERIES

DAILY Q & A SERIES

For Enrolment Join Telegram Channel: SIMPLIFIED_GS

o Also known as ‘Mahakavi Bharathiyar’.

o His strong sense of social justice drove him to fight for self-determination.

Involvement during Nationalist Period:

o After 1904, he joined the Tamil daily newspaper Swadesamitran.

This exposure to political affairs led to his involvement in the extremist wing of the Indian National Congress (INC) party.

o In order to proclaim its revolutionary ardour, Bharathi had the weekly newspaper named ‘India’ printed in red paper.

It was the first paper in Tamil Nadu to publish political cartoons.

He also published and edited a few other journals like “Vijaya”.

o Attended the annual sessions of INC and discussed national issues with extremist leaders like Bipin Chandra Pal, B.G. Tilak and V.V.S. Iyer.

His participation and activities in Benaras Session (1905) and Surat Session (1907) of the INC impressed many national leaders for his patriotic fervour.

o Published the sensational “Sudesa Geethangal” in 1908.

o Bharati’s reaction to the Russian Revolutions of 1917, in a poem entitled “Pudiya Russia” (“The New Russia”), offers a fascinating example of the poet’s political philosophy.

o He was forced to flee to Pondicherry (now Puducherry), a French colony, where he lived in exile from 1910 to 1919.

Q39) Lebanon is bordered by which of the following countries?

1. Israel

2. Qatar

3. Syria

a) 1 and 3 only

b) 1 and 2 only

c) 2 and 3 only

Page 24: DAILY Q & A SERIES

DAILY Q & A SERIES

For Enrolment Join Telegram Channel: SIMPLIFIED_GS

d) 1, 2 and 3

Ans : a)

Lebanon’s financial meltdown has swiftly worsened in the last month, with much of the country crippled by fuel shortages that have ignited country-wide security incidents.

Over two years, around 78 per cent of the Lebanese population has fallen into poverty. The World Bank says it is one of the sharpest depressions of modern times.

Early in the crisis, Lebanon defaulted on its massive pile of public debt, including $31 billion of Eurobonds that remain outstanding to creditors.

The currency has fallen by more than 90 per cent, demolishing purchasing power in a country dependent on imports. The banking system is paralysed.

The fuel shortages have led to confrontations at petrol stations, where motorists have to wait for hours and weapons have been drawn in melees over fuel. Fuel tankers have been hijacked.

Lebanon is a country in Western Asia. It is bordered by Syria to the north and east and Israel to the south, while Cyprus lies to its west across

the Mediterranean Sea.

Q40) Who had called Vivekananda the “maker of modern India.”

a) Jawahar lal Nehru

b) C R das

c) Subhas Chandra Bose

d) Rajendra Prasad

Ans: c)

On September 11, 1893, Swami Vivekananda delivered his famed speech at the ‘Parliament of the World’s Religions’, garnering a full two minute standing ovation and the moniker of ‘cyclonic monk of India’ .

This year marked the 128th anniversary of the historic Chicago Address of Swami Vivekananda.

Significance of this event:

The Chicago address had dwelt at length on Hinduism and Indian culture, and his words continue to remain resonant till date.

He became popular in the western world after his famous speech at the World’s Parliament of Religions.

He was considered a major force in the revival of Hinduism in India and bringing it to the status of major world religion in the late 19th century.

His address in the World “Parliament of Religions” at Chicago in 1893 drew the world’s attention to the ancient Indian philosophy of Vedanta.

Page 25: DAILY Q & A SERIES

DAILY Q & A SERIES

For Enrolment Join Telegram Channel: SIMPLIFIED_GS

About Swami Vivekananda:

He was a true luminary, credited with enlightening the western world about Hinduism. He was an ardent disciple of Sri Ramakrishna Paramahansa and a major force in the revival

of Hinduism in India. He pushed for national integration in colonial India, and his famous speech remains as the

one that he gave in Chicago in 1893 (Parliament of the World Religions). In 1984 the Government of India declared that 12 January, the birthday of Swami

Vivekananda, will be celebrated as National Youth Day.

Early life- contributions:

1. Born in Kolkata on January 12, 1863 in Kolkata, Swami Vivekananda was known as Narendra Nath Datta in his pre-monastic life.

2. He is known to have introduced the Hindu philosophies of Yoga and Vedanta to the West. 3. Netaji Subhas Chandra Bose had called Vivekananda the “maker of modern India.” 4. In 1893, he took the name ‘Vivekananda’ after Maharaja Ajit Singh of the Khetri State

requested him to do so. 5. He formed the Ramakrishna Mission in 1897 “to set in motion a machinery which will

bring noblest ideas to the doorstep of even the poorest and the meanest.” 6. In 1899, he established the Belur Math, which became his permanent abode. 7. He preached ‘neo-Vedanta’, an interpretation of Hinduism through a Western lens, and

believed in combining spirituality with material progress.

Books written by him:

‘Raja Yoga’, ‘Jnana Yoga’, ‘Karma Yoga’ are some of the books he wrote.

Q41) ‘REX MKII’ recently seen in the news, is associated with a) Indian Aerospace Industries b) China Aerospace Industries c) USA Aerospace Industries d) Israel Aerospace Industries

Ans : (d)

Recently, Israel Aerospace Industries unveiled a remote-controlled armed robot ‘REX MKII’, which can patrol battle zones, track infiltrators and open fire.

The use of robots in the war involves dealing with ethical dilemmas.

The proponents say that such semi-autonomous machines allow armies to protect their soldiers, while critics fear this marks another dangerous step toward robots making life-or-death

Page 26: DAILY Q & A SERIES

DAILY Q & A SERIES

For Enrolment Join Telegram Channel: SIMPLIFIED_GS

decisions.

o The robot can gather intelligence for ground troops, carry injured soldiers and supplies in and out of battle, and strike nearby targets.

o The Israeli military is currently using a smaller but similar vehicle called the Jaguar to patrol the border with the Gaza Strip.

o Unmanned ground vehicles are being increasingly used by other armies, including those of the United States, Britain and Russia.

Their tasks include logistical support, the removal of mines and firing weapons.

An alternative Smart Wall has been proposed to replace the physical and armed patrolling with advanced surveillance technology at the USA-Mexico border.

Q42 ) Consider the correct statement about Permafrost :

1) Permafrost is any ground that remains completely frozen - 32°F (0°C) or colder - for at least 3 years straight.

2) Permafrost covers about 25% of the land area of the globe. a) 1 only b) 2 only c) Both 1 and 2 d) Neither 1 nor 2

Ans: d)

According to the latest IPCC report, increasing global warming will result in reductions in Arctic permafrost and the thawing of the ground is expected to release greenhouse gases like methane and carbon dioxide.

Permafrost:

o Permafrost is any ground that remains completely frozen - 32°F (0°C) or colder - for at least two years straight.

o These permanently frozen grounds are most common in regions with high mountains and in Earth’s higher latitudes - near the North and South Poles.

o Permafrost covers about 15% of the land area of the globe.

o Although the ground is frozen, permafrost regions are not always covered in snow.

o Landscapes with large stretches of permafrost are often called tundra. The word tundra is a Finnish word referring to a treeless plain. Tundra is found at high latitudes and at high altitudes, where the permafrost has a very thin active layer.

Q43 ) T+1 and T+2 system , recently seen in the news , is related to

Page 27: DAILY Q & A SERIES

DAILY Q & A SERIES

For Enrolment Join Telegram Channel: SIMPLIFIED_GS

a) Current account Deficit

b) stock exchanges

c) Black Money

d) Environmental ecosystem

Ans : b)

Recently, Securities and Exchange Board of India (SEBI) allowed stock exchanges to start the T+1 system as an option in place of T+2 for completion of share transactions.

It has been introduced on an optional basis in a move to enhance liquidity.

SEBI is a statutory body established in 1992 in accordance with the provisions of the Securities and Exchange Board of India Act, 1992.

If the stock exchange opts for the T+1 settlement cycle for a scrip, it will have to mandatorily continue with it for a minimum 6 months.

A scrip is a substitute or alternative to legal tender that entitles the bearer to receive something in return.

Thereafter, if it intends to switch back to T+2, it will do so by giving one month’s advance notice to the market. Any subsequent switch (from T+1 to T+2 or vice versa) will be subject to a minimum period.

T+1 vs T+2 Settlement:

In T+2, if an investor sells shares, the settlement of the trade takes place in two working days (T+2) and the broker who handles the trade will get the money on the third day, but will credit the amount in the investor’s account only by the fourth day.

In effect, the investor will get the money only after three days.

In T+1, settlement of the trade takes place in one working day and the investor will get the money on the following day.

The move to T+1 will not require large operational or technical changes by market participants, nor will it cause fragmentation and risk to the core clearance and settlement ecosystem.

Benefits of T+1 Settlement:

Reduced Settlement Time: A shortened cycle not only reduces settlement time but also reduces and frees up the capital required to collateralise that risk.

Reduction in Unsettled Trade: It also reduces the number of outstanding unsettled trades at any instant, and thus decreases the unsettled exposure to Clearing Corporation by 50%.

Page 28: DAILY Q & A SERIES

DAILY Q & A SERIES

For Enrolment Join Telegram Channel: SIMPLIFIED_GS

The narrower the settlement cycle, the narrower the time window for a counterparty insolvency/bankruptcy to impact the settlement of a trade.

Reduction in Blocked Capital: Further, the capital blocked in the system to cover the risk of trades will get proportionately reduced with the number of outstanding unsettled trades at any point of time.

Reduction in Systemic Risks: A shortened settlement cycle will help in reducing systemic risk.

Q44) Consider the correct statement to the Raja Mahendra Pratap Singh :

1. He set up the first Provisional Government of India in Afghanistan in 1915, declaring himself as the

President.

2. He established the Executive Board of India in Japan in 1940.

a) 1 only

b) 2 only

c) Both 1 and 2

d) Neither 1 nor 2

Ans : c)

Prime Minister Narendra Modi will lay the foundation stone of Raja Mahendra Pratap Singh State University in Aligarh, Uttar Pradesh, on 14 September, 2021. The University is being established by the State Government in the memory and honour of the great freedom fighter, educationist and social reformer, Raja Mahendra Pratap Singh ji.

Born into a royal family in the Hathras district of Uttar Pradesh in 1886, Singh was a social reformer, freedom fighter and Marxist revolutionary.

An alumnus of the Muhammadan Anglo-Oriental Collegiate School, later called Aligarh Muslim University (AMU), he was active in the political arena from a young age.

He also participated in the 1911 Balkan War with his fellow students at the college. The freedom fighter set up the first Provisional Government of India in Afghanistan in 1915, declaring

himself as the President. He also declared a jihad against colonial rule, leading to the British announcing a bounty on his head.

Singh later fled to Japan, where he established the Executive Board of India in Japan in 1940. A firm believer in Mahatma Gandhi’s policy of non-violence, Singh also set up the Prem Maha Vidyalaya,

a polytechnic institution in his palace in Vrindavan. The social reformer was also a firm proponent of Panchayat Raj, believing it would reduce corruption and

put more power in the hands of the common man.

Page 29: DAILY Q & A SERIES

DAILY Q & A SERIES

For Enrolment Join Telegram Channel: SIMPLIFIED_GS

Singh, who called himself “the servant of the powerless and weak,” was nominated for the Noble Peace Prize in 1932.

Post-independence, he was elected to Parliament in 1957 after defeating then Jan Sangh (and later BJP) candidate Atal Bihari Vajpayee. Singh passed away in 1979.

Q45) India and which of the following country recently launched the “Climate Action and Finance

Mobilization Dialogue (CAFMD)”?

a) Spain

b) Sri Lanka

c) USA

d) UK

Ans: c)

India and the United States of America (USA) launched the “Climate Action and Finance Mobilization Dialogue (CAFMD)”.

The CAFMD is one of the two tracks of the India-U.S. Climate and Clean Energy Agenda 2030 partnership launched at the Leaders' Summit on Climate in April 2021, by Prime Minister Shri Narendra Modi and US President Mr. Joseph Biden.

The dialogue was formally launched by Union Minister of Environment, Forest and Climate Shri Bhupender Yadav and Mr. John Kerry, U.S. Special Presidential Envoy for Climate (SPEC), at an event held in New Delhi.

The dialogue will not only strengthen India-US bilateral cooperation on climate and environment but will also help to demonstrate how the world can align swift climate action with inclusive and resilient economic development, taking into account national circumstances and sustainable development priorities.

The launch was preceded by a bilateral meet where both sides discussed at length a wide range of climate issues relating to COP26, Climate Ambition, Climate Finance, Global Climate Initiatives including International Solar Alliance (ISA), Agriculture Innovation Mission for Climate (AIM4C). Q46) Which of the following state Legislative Assembly passed a Bill to dispense with the National

Entrance cum Eligibility Test (NEET) and allow admission to medical courses based on Class 12 marks

to “ensure social justice”?

a) Kerala

b) Andhra Pradesh

c) Karnataka

d) Tamil Nadu

Page 30: DAILY Q & A SERIES

DAILY Q & A SERIES

For Enrolment Join Telegram Channel: SIMPLIFIED_GS

Ans : d)

The Tamil Nadu Assembly passed a Bill to dispense with the National Entrance cum Eligibility Test (NEET) and allow admission to medical courses based on Class 12 marks to “ensure social justice”.

Chief Minister M K Stalin introduced the Bill based on the recommendation of the high-level committee led by retired judge AK Rajan, which submitted its report in July.

The Permanent Exemption Bill for NEET exempts medical aspirants in Tamil Nadu from taking NEET examination for admission to UG degree courses in Indian medicine, dentistry and homeopathy.

Instead, it seeks to provide admission to such courses on the basis of marks obtained in the qualifying examination, through “Normalisation methods”.

The Bill opposes NEET because it “undermined the diverse societal representation in MBBS and higher medical studies, favouring mainly the affordable and affluent sections of the society and thwarting the dreams of underprivileged social groups.

Q47) ‘Pusa Decomposer’ capsule has been developed by

a) Council of Scientific and Industrial Research

b) Indian Agriculture Research Institute

c) DRDO

d) ISRO

Ans : b)

Terming the bio-decomposer technique a “smashing success” at curbing stubble burning in the Capital, Chief Minister Arvind Kejriwal appealed to the Centre to ask the neighbouring States to use the same to prevent pollution.

The burning of paddy stubble left in the fields after harvest has been a cause of concern for the past several years as it contributes to air pollution in the northern Gangetic plains and its already polluted cities like Delhi.

It is a common practice in October and November across North West India, but primarily in Punjab, Haryana, and Uttar Pradesh to quickly clear crop residue from their fields before planting the rabi wheat crop.

To tackle the issue ‘Pusa Decomposer’ capsule has been developed by Indian Agriculture Research Institute (IARI).

‘Pusa Decomposer’ is essentially a fungi-based liquid solution that can soften hard stubble to the extent that it can be easily mixed with soil in the field to act as compost.

This would then rule out the need to burn the stubble, and also help in retaining the essential microbes and nutrients in soil that are otherwise damaged when the residue is burned.

There are seven strains of fungi that IARI has identified after research which help in rapid breakdown of hard stubble. These seven strains of fungi are packed into four capsules.

Q48) Consider the INCORRECT to “Operation Blue Freedom Triple World Records:

1. It is being undertaken by CLAW Global.

2. CLAW Global was set up in January 2019 by Major Vivek Jacob (retd), a Para Special Forces officer.

Page 31: DAILY Q & A SERIES

DAILY Q & A SERIES

For Enrolment Join Telegram Channel: SIMPLIFIED_GS

a) 1 only

b) 2 only

c) Both 1 and 2

d) Neither 1 nor 2

Ans : d)

A team of eight persons with disabilities created a new world record for the “largest number of people with disabilities” to scale the world’s highest battlefield, Siachen Glacier, by scaling up to the Kumar Post at an altitude of 15,632 ft.

The team with disabilities was trained and led by CLAW Global, a team of Special Forces veterans who had left the service due to disabilities.

This is the land world record expedition part of “Operation Blue Freedom Triple World Records” being undertaken by CLAW Global.

CLAW Global was set up in January 2019 by Major Vivek Jacob (retd), a Para Special Forces officer, who had to hang up his boots following a combat skydive injury after 14 years of service in the Army, with the aim of teaching life skills to adventurers and people with disabilities.

Q49) Saline Gargle RT-PCR technology Developed by

a) NEERI

b) DRDO

c) a & b

d) none of these

Ans : a)

The Nagpur-based National Environmental Engineering Research Institute (NEERI) has transferred the know-how of indigenously developed Saline Gargle RT-PCR technique to the Ministry of Micro, Small and Medium Enterprises (MSME) for commercialising it.

The Saline Gargle RT-PCR technology is simple, fast, cost-effective, patient-friendly and comfortable. It also provides instant test results and is well-suited for rural and tribal areas, given minimal

infrastructure requirements, the institute said. NEERI is an institute under the Council for Scientific and Industrial Research (CSIR). In the light of the prevailing pandemic situation and a possible third wave of COVID-19, CSIR-NEERI fast-

tracked the know-how transfer process to potential licensees for its wider dissemination across the nation.

Q50) Under which Union Ministry is the Department of Public Enterprises currently placed?

a) Ministry of Heavy Industries

Page 32: DAILY Q & A SERIES

DAILY Q & A SERIES

For Enrolment Join Telegram Channel: SIMPLIFIED_GS

b) Ministry of Finance

c) Ministry of Commerce

d) Ministry of Skill

Ans: b)

DPE was earlier under Ministry of Heavy Industries and Public Enterprises.

It has been brought under Finance minister in a bid to ease coordination regarding future disinvestment plans.

Finance ministry now comprises of six departments, after inclusion of DPE.

Other five departments are: 1. Department of Economic Affairs,

2. Department of Expenditure,

3. Department of Revenue,

4. Department of Investment & Public Asset Management and

5. Department of Financial Services.

Q51) Consider the correct statement about Groundswell report: 1) released by the World Bank indicated that climate change could force 216 million people

across six world regions to move within their countries by 2050. 2) First Groundswell Report was published in 2017. a) 1 only b) 2 only c) Both 1 and 2 d) Neither 1 nor 2

Ans : (a)

Recently, the updated Groundswell report released by the World Bank indicated that climate change could force 216 million people across six world regions to move within their countries by 2050.

Hotspots of internal climate migration can emerge as early as 2030 and continue to spread and intensify by 2050.

o First Groundswell Report: It was published in 2018 and used a robust and novel modeling approach to understand the scale, trajectory, and spatial patterns of future climate migration within countries, with a focus on three regions: Sub-Saharan Africa, South Asia, and Latin America.

o Second Groundswell Report: It builds on the first report, applying the same approach to three new regions: the Middle East and North Africa, East Asia and the Pacific, and Eastern Europe and Central Asia.

Page 33: DAILY Q & A SERIES

DAILY Q & A SERIES

For Enrolment Join Telegram Channel: SIMPLIFIED_GS

Qualitative analyses of climate-related mobility in countries of the Mashreq (i.e eastern part of the Arab World) and in Small Island Developing States (SIDS) are also provided.

The two reports’ combined findings provide, for the first time, a global picture of the potential scale of internal climate migration across the six World Bank regions.

Q52 ) Which of the following Countries included in lithium triangle :

1) Brazil

2) Argentina

3) Bolivia

4) Chile

5) Peru

a) 1,2 and 3

b) 2,3 and 4

c) 3,4 and 5

d) 2,3 and 5

Ans: b)

Various provinces of Argentina are building mining logistics nodes and access roads, lowering tax rates and rationalizing rules for the sector to attract investment in the Lithium metal.

Rising global lithium demand and surging prices have drawn increased interest in the so-called ‘lithium triangle’ that spans parts of Argentina, Bolivia and Chile.

Lithium has become the new 'white gold' as the demand for high performing rechargeable batteries is rising.

Q53 ) Consider the INCORRECT statement about Hindi Diwas :

1) The first Hindi Day was celebrated in 1953.

2) Article 351 pertains to ‘Directive for development of the Hindi language’.

3) Every year, 15th September is celebrated as Hindi Diwas in India.

a) 1 only

b) 1 and 3

c) 3 only

d) 2 only

Ans : c)

Every year, 14th September is celebrated as Hindi Diwas in India.

Page 34: DAILY Q & A SERIES

DAILY Q & A SERIES

For Enrolment Join Telegram Channel: SIMPLIFIED_GS

One of the reasons behind celebrating this day is to prevent the increasing trend of the English language in the nation and the neglect of Hindi.

o Hindi, written in the Devanagari script, was adopted as the official language of the Republic of India on 14th September, 1949.

Kaka Kalelkar, Maithili Sharan Gupta, Hazari Prasad Dwivedi, Seth Govindadas made important contributions to make Hindi the official language.

o The first Hindi Day was celebrated in 1953.

o Besides Hindi, English is the other official language (Article 343 of the Constitution).

o Hindi is also an eighth schedule language.

o Article 351 pertains to ‘Directive for development of the Hindi language’.

o Hindi is not a classical language.

World Hindi Day:

o It is observed on 10th January.

o Commemorates the anniversary of the first World Hindi Conference held in Nagpur on 10th January, 1975, which saw 122 delegates from 30 countries participating in it.

o First celebrated in 2006 by former Indian Prime Minister Dr. Manmohan Singh with an aim to promote Hindi language across the world.

o The World Hindi Secretariat building was inaugurated in Mauritius in 2018.

Q54) Consider the correct statement about BRICS Young Scientists Forum:

1) The BRICS-YSF summits were first hosted by India in 2019.

2) Young innovator prize has been one of the focuses of the BRICS-YSF

and the award is supported by the DST, Government of India.

a) 1 only

b) 2 only

c) Both 1 and 2

d) Neither 1 nor 2

Ans : b)

Dr Renu Swarup, Secretary Department of Science and Technology highlighted the importance of cooperation, collaboration and connections for science to move forward, at the BRICS Young Scientists Forum.

The BRICS-YSF summits were first hosted by India in 2016, followed by China in 2017, South Africa in 2018, Brazil in 2019, and Russia in 2020.

This year the four-day conclave started on 13 September 2021 would end on 16 September. The best young scientist with an idea on innovation would be awarded at the BRICS-YSF 2021.

Page 35: DAILY Q & A SERIES

DAILY Q & A SERIES

For Enrolment Join Telegram Channel: SIMPLIFIED_GS

Young innovator prize has been one of the focuses of the BRICS-YSF and the award is supported by the DST, Government of India. The next conclave would be organized by China in 2022.

Q55) Which of the following pairs is/are correctly matched?

Tribe State/U.T

1. Gujjar-Bakerwals Jammu & Kashmir

2. Gaddi-Sippis Kerala

3. Kond Karnataka

Select the correct answer using the code given below:

a) 1 only

b) 1 and 2 only

c) 2 and 3 only

d) 1, 2 and 3

Ans: a)

After a long delay, the Jammu and Kashmir government has decided to implement the Forest Rights Act, 2006.

This will elevate the socio-economic status of a sizeable section of the 14 lakh population of tribals and nomadic communities, including Gujjar-Bakerwals and Gaddi-Sippis in the Union Territory.

In the past few years, there were growing cases of eviction of tribals from forest land in parts of the Kashmir Valley and Jammu region. The government termed them “illegal encroachers” but the regional parties accused the Forest Department of acting above the law in these cases.

The Union Territory saw the arbitrary demolition of Gujjar and Bakerwal houses during a forest reclamation drive in contravention of the Forest Rights Act of 2006.

Javaid Rahi, a tribal rights activist, termed the J&K government’s move a “historic occasion”. Kond is one of the largest tribe in Odisha who worship hills, nature, and streams.

Q56) Which of the following institute recently launched ‘Project Udaan’?

Page 36: DAILY Q & A SERIES

DAILY Q & A SERIES

For Enrolment Join Telegram Channel: SIMPLIFIED_GS

a) IIT Bombay

b) IIT Delhi

c) IIT Madras

d) IIT Roorkee

Ans : a)

With an aim to break the language barrier that many students face when they join institutes of higher education, IIT-Bombay launched ‘Project Udaan’.

It enables translation of textbooks and other study material related to engineering and other streams. Prof. Ganesh and his team have built an Artificial Intelligence-based translation ecosystem, which can

help translate engineering textbooks and learning materials in one-sixth the time it would take for a team consisting of domain and linguistic experts working manually.

Translation of text books in other educational domains will be taken up in due course.

Q57) Consider the correct statement to green hydrogen :

1) Hydrogen when produced by electrolysis using renewable energy is known as Green Hydrogen.

2) Green Hydrogen can act as an energy storage option, which would be essential to meet intermittencies (of renewable energy) in the future.

a) 1 only b) 2 only c) Both 1 and 2 d) Neither 1 nor 2

Ans : c)

Power and New and Renewable Energy Minister RK Singh has urged US companies to participate in the bids for green hydrogen and electrolysers in the upcoming months.

Challenges:

The path for green hydrogen in the country is not clear and at the moment, production of green hydrogen is slightly more expensive than grey hydrogen.

What is green hydrogen?

Hydrogen when produced by electrolysis using renewable energy is known as Green Hydrogen which has no carbon footprint.

Page 37: DAILY Q & A SERIES

DAILY Q & A SERIES

For Enrolment Join Telegram Channel: SIMPLIFIED_GS

Significance of Green Hydrogen:

Green hydrogen energy is vital for India to meet its Nationally Determined Contribution (INDC) Targets and ensure regional and national energy security, access and availability.

Green Hydrogen can act as an energy storage option, which would be essential to meet intermittencies (of renewable energy) in the future.

In terms of mobility, for long distance mobilisations for either urban freight movement within cities and states or for passengers, Green Hydrogen can be used in railways, large ships, buses or trucks, etc.

Applications of green hydrogen:

Green Chemicals like ammonia and methanol can directly be utilized in existing applications like fertilizers, mobility, power, chemicals, shipping etc.

Green Hydrogen blending up to 10% may be adopted in CGD networks to gain widespread acceptance.

Benefits:

It is a clean-burning molecule, which can decarbonize a range of sectors includin g iron and steel, chemicals, and transportation.

Renewable energy that cannot be stored or used by the grid can be channelled to produce hydrogen.

Q58) Which of the following organization, China said “Asian NATO” ?

a) SAARC

b) QUAD

c) ASEAN

d) SCO

Ans : b)

The quadrilateral security dialogue includes Japan, India, United States and Australia.

All four nations find a common ground of being the democratic nations and common interests of unhindered maritime trade and security.

What are China’s views on the Quad?

There is a general understanding that the Quad would not take on a military dimension against any country. The strategic community in China, nevertheless, had branded it an emerging “Asian NATO”.

Notably, Japanese PM Shinzo Abe’s “Confluence of Two Seas” address to the Indian Parliament gave a fresh impetus to the Quad concept. This recognised the economic rise of India.

Page 38: DAILY Q & A SERIES

DAILY Q & A SERIES

For Enrolment Join Telegram Channel: SIMPLIFIED_GS

Q59) Which of the following country has the unique distinction of being the only country in the world producing all Five major types of silk?

a) India

b) China

c) Brazil

d) Thailand

Ans : a)

India has the unique distinction of being the only country producing all the five kinds of silk namely,

Mulberry, Eri, Muga, Tropical Tasar and Temperate Tasar. Among them, mulberry silk is the most

popular variety, which contributes around 79% of the country's silk production.

Q60) Which of the following is NOT one of the nine pillars of Digital India?

a) e-Kranti - Electronic Delivery of Services.

b) Electronics Manufacturing

c) Affordable Internet availability

d) Universal access to mobile connectivity

Ans: c)

Nine pillars of Digital India

1) Broadband Highways

2) Universal Access to Mobile Connectivity

3) Public Internet Access Programme

4) e-Governance: Reforming Government through Technology

5) e-Kranti - Electronic Delivery of Services

6) Information for All

7) Electronics Manufacturing

8) IT for Jobs

9) Early Harvest Programmes.

Q61) Recently, which organization ave launched the Shoonya Campaign ? 1) NITI Aayog 2) Rocky Mountain Institute 3) World Bank

Page 39: DAILY Q & A SERIES

DAILY Q & A SERIES

For Enrolment Join Telegram Channel: SIMPLIFIED_GS

4) NATO

a) 1 only

b) 1 and 2

c) 2 and 3

d) 2,3 and 4

Ans : (b)

Recently, NITI Aayog and Rocky Mountain Institute (RMI) and RMI India’s have launched the Shoonya Campaign.

It is an initiative to promote zero-pollution delivery vehicles by working with consumers and industry.

RMI is an independent nonprofit organization founded in 1982.

Shoonya Campaign:

o Electric Deliveries: The campaign aims to accelerate adoption of Electric Vehicles (EVs) in the urban deliveries segment and create consumer awareness about the benefits of zero-pollution delivery.

o Shoonya Brand: A corporate branding and certification programme is being launched to recognise and promote industry’s efforts towards transitioning to EVs for final-mile deliveries.

It will help e-commerce companies to distinguish their offerings from those of their competitors.

o Online Tracking Platform : An online tracking platform will share the campaign’s impact through data such as vehicle kilometers electrified, carbon savings, criteria pollutant savings and other benefits from clean delivery vehicles.

Q62 ) A recent study in Bihar has found that Arsenic contamination not only in groundwater but in the food chain as well.

The research study was a part of the Project Nature and Nurture in Arsenic Induced Toxicity of Bihar jointly funded by

1) British Council 2) Department of Science and Technology , India 3) Niti Aayog 4) RBI 5) IMF

a) 1 and 3 b) 2,4 and 5 c) 1 and 2 d) 1,2,3 and 4

Ans: c)

Page 40: DAILY Q & A SERIES

DAILY Q & A SERIES

For Enrolment Join Telegram Channel: SIMPLIFIED_GS

A recent study in Bihar has found that Arsenic contamination not only in groundwater but in the food chain as well.

The research study was a part of the Project Nature and Nurture in Arsenic Induced Toxicity of Bihar jointly funded by the British Council in the United Kingdom and Department of Science and Technology in India.

o Food Chain Contamination:

Arsenic has found its way into the food chain - mainly rice, wheat and potato.

Arsenic contamination in groundwater has been a growing concern in several parts of the country.

Arsenic is present in the groundwater as it is used on a large scale for irrigation by farmers. That is how it finds its way into the food chain as well.

o Food vs Water Contamination:

The food had more arsenic content than drinking water, even when arsenic levels in drinking water was above the World Health Organization (WHO) provisional guide value of 10 micrograms per litre (μg/L).

The concentration was higher in cooked rice compared to raw rice.

Arsenic Poisoning:

Page 41: DAILY Q & A SERIES

DAILY Q & A SERIES

For Enrolment Join Telegram Channel: SIMPLIFIED_GS

It can get into the human body through drinking water as well as eating food that has been contaminated with arsenic.

Arsenicosis is the medical word for arsenic poisoning, which occurs due to accumulation of large amounts of arsenic in the body.

It leads to adverse health effects through inhibition of essential enzymes, which ultimately leads to death from multi-system organ failure.

Long-term exposure to arsenic from drinking-water and food can cause cancer and skin lesions. It has also been associated with cardiovascular disease and diabetes.

In utero and early childhood exposure has been linked to negative impacts on cognitive development and increased deaths in young adults.

Q63 ) Consider the correct statement about ‘Black Tigers :

1) Similipal Tiger Reserve is the only tiger habitat in the world with melanistic tigers.

2) Black Tigers are a rare colour variant of the tiger and are not a distinct species or geographic subspecies.

a) 1 only

b) 2 only

c) Both 1 and 2

d) Neither 1 nor 2

Ans : c)

Recently, scientists have unraveled the mystery behind Odisha’s ‘Black Tigers’ in Similipal Tiger Reserve (STR)

The STR is the only tiger habitat in the world with melanistic tigers, which have broad black stripes running across their bodies and thicker than those seen on normal tigers.

Page 42: DAILY Q & A SERIES

DAILY Q & A SERIES

For Enrolment Join Telegram Channel: SIMPLIFIED_GS

o Black Tigers are a rare colour variant of the tiger and are not a distinct species or geographic subspecies.

o The coat colouration and patterning that make the wild cats appear dark boil down to a single mutation in the Transmembrane Aminopeptidase Q (Taqpep) gene.

o The abnormally dark or black coat in such tigers is termed pseudo melanistic or false coloured.

o If you pick any tiger from Similipal, the chance that it carries the mutant gene is almost 60%.

Factors for Occuring Black Colour:

o Due to Geographic Isolation, genetically related individuals have been mating with each other for many generations in Similipal, leading to inbreeding.

It should be noted that this has important implications for tiger conservation as such isolated and inbred populations are prone to extinction over even short periods of time.

Q64) Consider the correct statement about Exercise PEACEFUL MISSION:

1) It is a Multilateral Exercise, which is conducted biennially as part of military diplomacy

between Shanghai Cooperation Organisation (SCO) member states.

2) The Shanghai Cooperation Organization (SCO) is an intergovernmental organization founded

in Shanghai on 2002.

a) 1 only

b) 2 only

c) Both 1 and 2

d) Neither 1 nor 2

Ans : a)

Joint Counter Terrorism Exercise PEACEFUL MISSION is a Multilateral Exercise, which is conducted biennially as part of military diplomacy between Shanghai Cooperation Organisation (SCO) member states.

The 6th edition of Exercise PEACEFUL MISSION is being hosted by Russia in the Orenburg Region of South West Russia from 13 to 25 September 2021.

The aim of the exercise is to foster close relations between SCO member states and to enhance abilities of the military leaders to command multi-national military contingents.

The Shanghai Cooperation Organization (SCO) is an intergovernmental organization founded in Shanghai on 15 June 2001.

The SCO currently comprises eight Member States (China, India, Kazakhstan, Kyrgyzstan, Russia, Pakistan, Tajikistan and Uzbekistan) and four Observer States interested in acceding to full membership (Afghanistan, Belarus, Iran, and Mongolia).

Q65) Consider the INCORRECT statement about Hybodonts :

1) Hybodont sharks have been reported for the first time from the Jurassic rocks of the Jaisalmer

region of Rajasthan.

Page 43: DAILY Q & A SERIES

DAILY Q & A SERIES

For Enrolment Join Telegram Channel: SIMPLIFIED_GS

2. Hybodonts, an extinct group of sharks, was a dominant group of fishes in both marine and fluvial

environments during the Triassic and early Jurassic time.

a) 1 only

b) 2 only

c) Both 1 and 2

d) Neither 1 nor 2

Ans: d)

In a rare discovery, teeth of new species of hybodontshark of Jurassic age have been reported for the first time from Jaisalmer by a team of officers from the Geological Survey of India (GSI), Western Region, Jaipur.

Hybodont sharks have been reported for the first time from the Jurassic rocks (approximately, between 160 and 168 million-years-old) of the Jaisalmer region of Rajasthan.

Hybodonts, an extinct group of sharks, was a dominant group of fishes in both marine and fluvial environments during the Triassic and early Jurassic time.

However, hybodont sharks started to decline in marine environments from the Middle Jurassic onwards until they formed a relatively minor component of open-marine shark assemblages.

Hybodonts finally became extinct at the end of the Cretaceous time 65 million years ago. Significantly, the newly discovered crushing teeth from Jaisalmer represent a new species named by the

research team as Strophodus jaisalmerensis. The genus Strophodus has been identified for the first time from the Indian subcontinent and is only the

third such record from Asia, the other two being from Japan and Thailand. The new species has recently been included in the Shark references.com, an international platform

operating in association with International Union for Conservation of Nature (IUCN), Species Survival Commission (SSC), and Germany. Q66) Which of the following statement is not correct with respect to Tarballs?

a)

Tarballs are dark-coloured, sticky balls of oil that form when crude oil floats on the ocean

surface.

b) They are transported from the open sea to the shores by sea currents and waves.

c) Tarballs stick to the cleaning machinery and are very difficult to wash off.

d) None of the above.

Ans : d)

In this month alone, the Brihanmumbai Municipal Corporation (BMC) has removed over 20,000 kg of tarballs from Juhu and Versova beaches.

Page 44: DAILY Q & A SERIES

DAILY Q & A SERIES

For Enrolment Join Telegram Channel: SIMPLIFIED_GS

Tarballs are dark-coloured, sticky balls of oil that form when crude oil floats on the ocean surface. Tarballs are formed by weathering of crude oil in marine environments.

They are transported from the open sea to the shores by sea currents and waves. Some of the balls are as big as a basketball while others are smaller globules. Tarballs stick to the cleaning machinery and are very difficult to wash off. It is suspected that the oil comes from the large cargo ships in the deep sea and gets pushed to the shore

as tarballs during monsoon due to wind speed and direction. All the oil spilt in the Arabian sea eventually gets deposited on the western coast in the form of tarballs in

the monsoon season when wind speed and circulation pattern favour transportation of these tarballs. Q67) In which city, PM Modi has laid foundation stone of Raja Mahendra Pratap Singh University?

a) Aligarh b) Meerut c) Jaipur d) Varanasi

Ans : a)

Prime Minister Narendra Modi has laid the foundation stone of Raja Mahendra Pratap Singh State University in Aligarh. The university is being established by the Uttar Pradesh government in the memory and honour of Raja Mahendra Pratap Singh, freedom fighter, educationist and social reformer. The university will provide affiliation to 395 colleges of the Aligarh division.

Q68) Recently, which airport has been declared as a Customs notified airport by The Central Board of

Indirect Taxes and Customs?

a) Kangra Airport b) Pakyong Airport c) Swami Vivekananda International Airport d) Kushinagar Airport

Ans : d)

The Central Board of Indirect Taxes and Customs (CBIC) has declared Kushinagar Airport as a Customs notified airport. This would also facilitate international passenger movements including that of Buddhist pilgrims.

Q69) Which country has commenced the world’s biggest trial of a blood test where they will detect

more than 50 types of cancer?

a)China b)Russia c)France d) United Kingdom

Ans : d)

Page 45: DAILY Q & A SERIES

DAILY Q & A SERIES

For Enrolment Join Telegram Channel: SIMPLIFIED_GS

The United Kingdom’s state-run National Health Service (NHS) has commenced the world’s biggest trial of a blood test where they will detect more than 50 types of cancer. The world’s largest trial of Grail Inc’s flagship Galleri blood test will find out different types of cancer even before the symptoms appear.

Q70) Consider the correct statement about M Visvesvaraya:

1) serving as the Diwan of Mysore.

2) he was awarded ‘Knight’ as a commander of the British Indian Empire by King George V.

3) Every year on September 15, India along with Sri Lanka and Tanzania celebrate National Engineer’s day to recognise and honour the achievements of the great engineer Mokshagundam Visvesvaraya.

a) 1 only

b) 1 and 2

c) 1 and 3

d) 1,2 and 3

Ans: d)

Every year on September 15, India along with Sri Lanka and Tanzania celebrate National Engineer’s day to recognise and honour the achievements of the great engineer Mokshagundam Visvesvaraya.

The day is observed to commemorate the great work of engineers and to encourage them for improvement and innovation.

This year marks the 160th birth anniversary of M Visvesvaraya who was born on September 15, 1861, in the Muddenahalli village of Karnataka.

About Sir M Visvesvaraya:

An expert in irrigation techniques and flood disaster management, Sir MV was not just a great civil engineer but also served as the 19th Diwan of Mysore from 1912 to 1919.

While serving as the Diwan of Mysore, in 1915 he was awarded ‘Knight’ as a commander of the British Indian Empire by King George V.

Q71) Recently, National Crime Records Bureau (NCRB) released the Crime in India Report 2020. Select the correct statement :

1) Communal riots registered an increase of 96% in 2020 over the previous year. 2) Mumbai is the most unsafe city for women.

a) 1 only b) 2 only c) Both 1 and 2 d) Neither 1 nor 2

Page 46: DAILY Q & A SERIES

DAILY Q & A SERIES

For Enrolment Join Telegram Channel: SIMPLIFIED_GS

Ans : (a)

Recently, National Crime Records Bureau (NCRB) released the Crime in India Report 2020.

While 2020, a year marked by months of a national lockdown due to the pandemic saw fewer traditional crimes such as those committed against women and children, among others, it witnessed a big spike in civil conflicts.

Riots (Civil Conflicts):

Communal riots registered an increase of 96% in 2020 over the previous year.

Delhi Police alone registered the highest i.e. 520 cases of communal riots in the year and Uttar Pradesh (UP) did not record a single case of communal violence in 2020.

Caste riots saw an increase of close to 50%, agrarian riots 38% and riots during ‘andolan/morcha’ increased by 33%.

Traditional Crimes:

The number of cases registered for crimes against women, children and senior citizens, theft, burglary, robbery and dacoity, among others, dropped by about 2 lakh.

Murder registered a marginal increase of 1% even as offences falling under the category of “violent crimes” decreased by 0.5%.

Delhi is the most unsafe city for women. More than 10,093 cases of crimes against women were registered in the capital in 2020.

Environment Related Offences:

Cases under the ‘environment-related offences’ category increased by 78.1% in the country in 2020.

Cyber Crime:

The rate of cyber crime (incidents per lakh population) also increased from 3.3% in 2019 to 3.7% in 2020.

Offences Against State:

The year also saw a significant drop in cases related to Offences against the State, with a drop of 27% over 2019.

UP, however, was the only major state to record an increase in this category, mostly due to the large number of ‘Damage to Public Property’

Page 47: DAILY Q & A SERIES

DAILY Q & A SERIES

For Enrolment Join Telegram Channel: SIMPLIFIED_GS

cases registered by the state, many of them during the anti- CAA (Citizenship (Amendment) Act, 2019) protests.

Offences against the State’ include cases related to sedition and waging war against the nation, those falling under provisions of the Unlawful Activities Prevention Act (UAPA) 1967, Official Secrets Act 1923 and Damage to Public Property Act 1954.

Q72 ) Sufficiency Economy Philosophy is a part of fundamental administration policy of

a) China

b) Fiji

c) Thailand

d) UAE

Ans: C)

Thailand believes that its homegrown development approach of Sufficiency Economy Philosophy (SEP) can serve as an alternative approach to achieving the Sustainable Development Goals (SDGs).

o It is an innovative approach to development designed for practical application over a wide range of problems and situations.

It is also part of the fundamental administration policy of the state (Thailand).

It was introduced in Thailand after the Asian financial crisis in 1997.

o It is a philosophy that guides one’s inner thinking to immunize oneself from external shocks and can be applied in any setting and at any levels.

Q73 ) Baijayant Panda committee, recently seen in news, is related to:

a)

comprehensive review of National Cadet Corps (NCC) in order to make it more relevant in

changed times.

b) decide the appropriate level of reserves that the RBI should hold.

c) finance Infrastructure through PPP model.

d) None of the above

Ans : a)

Ministry of Defence has constituted a High Level Expert Committee, under the chairmanship of former Member of Parliament Shri Baijayant Panda, for a comprehensive review of National Cadet Corps (NCC) in order to make it more relevant in changed times.

Page 48: DAILY Q & A SERIES

DAILY Q & A SERIES

For Enrolment Join Telegram Channel: SIMPLIFIED_GS

The Terms of Reference of the Committee, inter-alia, broadly provide for o suggesting measures which can empower NCC cadets to contribute more effectively towards nation

building and national developmental efforts in various sectors; o propose measures for gainful engagement of its Alumni for betterment of the organisation as a whole

and to study/recommend best practices of similar international youth organisations for inclusion in the NCC curriculum.

Q74) Which of the following organization issues India’s first-ever Euro Green Bond?

a) Power Finance Corporation Ltd (PFC)

b) LIC

c) ONGC

d) None of the above

Ans : a)

PFC issues India’s first-ever Euro Green Bond.

Power Finance Corporation Ltd (PFC), the leading NBFC in power sector, has successfully issued its maiden Euro 300 million 7-year Euro Bond issuance on 13.09.2021.

The pricing of 1.841% achieved is the lowest yield locked in by an Indian Issuer in the Euro markets. It is the first ever Euro denominated Green bond issuance from India. Moreover, it is the first ever Euro

issuance by an Indian NBFC and the first Euro bond issuance from India since 2017.

Q75) Consider the correct to All India Debt & Investment Survey, consider the following statements:

1. It was conducted by the National Statistical Office (NSO).

2. The number of indebted households have risen sharply in rural India, with the average debt

shooting up by 84% between 2012 and 2018.

a) 1 only

b) 2 only

c) Both 1 and 2

Page 49: DAILY Q & A SERIES

DAILY Q & A SERIES

For Enrolment Join Telegram Channel: SIMPLIFIED_GS

d) Neither 1 nor 2

Ans: c)

The All India Debt & Investment Survey conducted by the National Statistical Office (NSO) was released recently.

The number of indebted households have risen sharply in rural India, with the average debt shooting up by 84% between 2012 and 2018, and COVID-19 has likely further doubled all households’ borrowings by 2021.

Rural households’ average debt grew from ₹32,522 in 2012 to ₹59,748 by June 2018, while urban households’ average debt increased by 42% in the same period to little more than ₹1.20 lakh.

The number of households in debt as measured by the incidence of indebtedness among surveyed households, had risen to 35% in rural India from 31.4% in the previous survey, while it remained static at 22.4% for urban households. Q76) Which of the following country is part of new trilateral security partnership (AUKUS) for the

Indo-Pacific?

1. India

2. U.K

3. Australia

a) 1 only

b) 1 and 2 only

c) 2 and 3 only

d) 1, 2 and 3

Ans : c)

The USA administration announced a new trilateral security partnership for the Indo-Pacific between Australia, the U.K. and the U.S. (AUKUS).

The security grouping AUKUS will focus on advancing strategic interests in the Indo-Pacific region A central feature of the partnership would involve a trilateral 18-month effort to help Australia acquire

nuclear-powered submarines which are quieter, more capable (than their conventional counterparts) and can be deployed for longer periods, needing to surface less frequently.

Page 50: DAILY Q & A SERIES

DAILY Q & A SERIES

For Enrolment Join Telegram Channel: SIMPLIFIED_GS

The partnership would also involve a new architecture of meetings and engagements between the three countries and also cooperation across emerging technologies (applied AI, quantum technologies and undersea capabilities).

Australia has felt increasing pressure from an assertive China, like other countries in the region, and has sought to strengthen its partnerships with India, the U.S. and the U.K., including through ‘plurilateral’ forums.

However, as in the case of the Quad, the U.S. officials denied the partnership was a response to China.

Q77) Which organization has launched a report titled ‘Reforms in Urban Planning Capacity in India?’

a) National Development Council b) NITI Aayog c) Ministry of Statistics and Programme Implementation d) Ministry of Housing and Urban Affairs

Ans : b)

NITI Aayog has launched a report on measures to ramp up urban planning capacity in India. The report is titled ‘Reforms in Urban Planning Capacity in India.’ The report has been developed by NITI Aayog, in consultation with concerned ministries and eminent experts in the domain of urban and regional planning. It presents a condensed outcome of the extensive deliberations and consultations conducted over a period of 9 months.

Q78) Recently, which organization has launched four amateurs on its first ever Earth-circling trip?

a) Blue Origin b) Virgin Galactic c) SpaceX d) Lockheed Martin

Ans : c)

SpaceX has launched four amateurs on its first ever private flight on September 15, 2021. SpaceX first ever “Earth-circling trip” was started on boarding two contest winners, a health care worker as well as their rich sponsor. This is the most ambitious leap in space tourism. This was the first time any rocket blasted off toward orbit with an all-amateur crew and no professional astronauts. Two men and two women are looking forward to spend three days on Dragon capsule circling the world. They will be circling the Earth on an unusually high orbit at 160 km higher than the International Space Station. Their flight will splash down off the Florida coast by September 19th Dragon capsule aimed for an altitude of 575 km, beyond the Hubble Space Telescope.

Q79) Consider the INCORRECT statement about National Financial Reporting Authority (NFRA) :

1) NFRA was constituted on 1st October, 2020 under section 132 (1) of the Companies Act, 2013.

2) Monitor and enforce compliance with accounting standards and auditing standards.

a) 1 only

b) 2 only

Page 51: DAILY Q & A SERIES

DAILY Q & A SERIES

For Enrolment Join Telegram Channel: SIMPLIFIED_GS

c) Both 1 and 2

d) Neither 1 nor 2

Ans : a)

National Financial Reporting Authority (NFRA) Chairperson has pitched for a ‘standalone legislation’ for the regulator in the interests of autonomy.

He also demanded that all necessary penal provisions relating to financial reporting should be consolidated and vested with it.

Why this is necessary?

Currently, the NFRA may take action against auditors for professional misconduct but when it came to other functionaries of a company who have the responsibility for financial reporting, penal powers continue to be vested with the Centre. A standalone legislation will allow for integrated regulation of all participants in the financial reporting system.

About NFRA:

National Financial Reporting Authority (NFRA) was constituted on 1st October, 2018 under section 132 (1) of the Companies Act, 2013.

Composition:

The Companies Act requires the NFRA to have a chairperson who will be appointed by the Central Government and a maximum of 15 members.

Functions and Duties:

1. Recommend accounting and auditing policies and standards to be adopted by companies for approval by the Central Government;

2. Monitor and enforce compliance with accounting standards and auditing standards; 3. Oversee the quality of service of the professions associated with ensuring compliance with

such standards and suggest measures for improvement in the quality of service; 4. Perform such other functions and duties as may be necessary or incidental to the aforesaid

functions and duties.

Q80) Who has become the first woman excavation engineer to work in an open cast mine at Central

Coalfields Ltd?

a) Akanksha Kumari b) Shivangi Singh c) Bhawana Kanth d) Shivani Meena

Ans: d)

Page 52: DAILY Q & A SERIES

DAILY Q & A SERIES

For Enrolment Join Telegram Channel: SIMPLIFIED_GS

Shivani Meena, an IIT Jodhpur alumna, has become the first woman excavation engineer to work in an

open cast mine at Coal India arm CCL (Central Coalfields Ltd). She will be posted at CCL’s Rajrappa

project, one of the important enterprises of the coal mining behemoth. Until now the position belonged

to men. The feat follows another milestone when Akanksha Kumari became Coal India’s first woman

mining engineer to work in an underground mine at CCL’s Churi facility in North Karanpura area in

Jharkhand.

Q81) Which organization has launched the Planetarium Innovation Challenge?

a) MyGov India b) ISRO c) NITI Aayog d) Birla Planetarium

Ans : (a)

The “Planetarium Innovation Challenge” has been launched by MyGov India for tech entrepreneurs and Indian start-ups. This Planetarium challenge was launched with the primary aim of bringing together local Start-ups and tech firms that have potential to build indigenous planetariums system software. This system software must be built by using latest technologies such as Virtual Reality (VR), Augmented Reality (AR) and Merged Reality (MR).

Q82 ) Recently, RBI has given its nod to which bank to form a special committee of directors in order

to oversee the operations?

a) Equitas Small Finance Bank b) IDFC First Bank c) Yes Bank d) Ujjivan Small Finance Bank

Ans: d)

Reserve Bank of India has given its approval to the Ujjivan Small Finance Bank (SFB) to form a special committee of directors so that they can oversee the operations in absence of a CEO and MD. This Special Committee of Directors will comprise of three independent directors as its members.

Q83 ) Consider the correct statement about Urbanisation in India:

1) Over 75% of the urban population of the country is in 10 States.

2) NCT of Delhi, Daman and Diu, Chandigarh, and Lakshadweep, show above 75% urbanisation.

a) 1 only

b) 2 only

c) Both 1 and 2

d) Neither 1 nor 2

Page 53: DAILY Q & A SERIES

DAILY Q & A SERIES

For Enrolment Join Telegram Channel: SIMPLIFIED_GS

Ans : c)

Recently, NITI Aayog has launched the report titled ‘Reforms in Urban Planning Capacity in India’.

Urbanisation in India:

o Urbanisation Level (National):

India’s population stood at 1210 million in 2011, with an urbanisation level of 31.1% (Census of India 2011).

Urbanisation is an increase in the number of people living in towns and cities.

The distribution of urban centres and the pace of urbanisation is not uniform across the country.

Over 75% of the urban population of the country is in 10 States: Maharashtra, Uttar Pradesh, Tamil Nadu, West Bengal, Andhra Pradesh, Gujarat, Karnataka, Madhya Pradesh, Rajasthan, and Kerala.

o State-wise Scenario:

Above National Average: Goa, Tamil Nadu, Kerala, Maharashtra, and Gujarat have attained over 40% urbanisation.

Below National Average: Bihar, Odisha, Assam, and Uttar Pradesh continue to be at a lower level of urbanisation than the national average of 31.1%.

Page 54: DAILY Q & A SERIES

DAILY Q & A SERIES

For Enrolment Join Telegram Channel: SIMPLIFIED_GS

Union Territories: NCT of Delhi, Daman and Diu, Chandigarh, and Lakshadweep, show above 75% urbanisation.

Q84) Recently, the Archaeological Survey of India (ASI) discovered remains of an ancient temple dating back to the Gupta period (5th century) in

a) Bihar b) Odisha c) Uttar Pradesh d) Madhya Pradesh

Ans : c)

Recently, the Archaeological Survey of India (ASI) discovered remains of an ancient temple dating back to the Gupta period (5th century) in Bilsarh village in Uttar Pradesh’s Etah district.

The Bilsarh site was declared ‘protected’ in 1928 by the ASI.

o Two pillars were excavated on which there is an inscription about Kumaragupta I, the powerful ruler of Gupta dynasty, in ‘sankh lipi’ (conch script or shell script) typical of the 5th century AD.

The Guptas were the first to build structural temples, distinctly different from the ancient rock-cut temples.

o The inscription was deciphered as Mahendraditya which was the title of king Kumargupta I who had even conducted ashwamedh yagya during his rule.

The statue of a horse having similar inscription is at the state museum in Lucknow.

Ashvamedha Yagya is a horse sacrifice ritual followed by the Srauta tradition of Vedic religion.

o The discovery becomes significant since only two other structural temples from the Gupta age have been found so far — Dashavatara Temple (Deogarh) and Bhitargaon Temple (Kanpur Dehat).

Q85) Consider the correct statement about SPIN Scheme:

1) Khadi and Village Industries Commission (KVIC) launched the SPIN

(Strengthening the Potential of India) scheme to make potters self-

sustainable.

2) aimed at sustainable development by creating local self-

employment which is aligned with the Prime Minister’s

commitment of “Job to Every Hand” (Har Hath Me Kaam).

Page 55: DAILY Q & A SERIES

DAILY Q & A SERIES

For Enrolment Join Telegram Channel: SIMPLIFIED_GS

a) 1 only

b) 2 only

c) Both 1 and 2

d) Neither 1 nor 2

Ans: c)

Recently, the Khadi and Village Industries Commission (KVIC) launched the SPIN (Strengthening the Potential of India) scheme to make potters self-sustainable.

SPIN Scheme:

o About:

Under SPIN, KVIC will facilitate potters to get easy loans from banks that will help the potters to diversify their activities and enhance their income.

o Objective:

It is aimed at sustainable development by creating local self-employment which is aligned with the Prime Minister’s commitment of “Job to Every Hand” (Har Hath Me Kaam).

o Features:

It is a no-subsidy program.

KVIC facilitates potters to get bank loans under Pradhan Mantri Shishu Mudra Yojana.

No financial burden on the exchequer.

Beneficiaries can repay the loans in easy installments.

Q86) Recently, a petition has been submitted by some activists to the US Food and Drug

Administration (FDA) to reclassify toxic 3 Os used in sunscreen. 3 Os ia a

a) Oxybenzone, Octinoxate and Octocrylene

b) Oxybenzone, Osazone and Octocrylene

c) Osazone, Oxybenzone, and Ozone

d) None of these

Ans : a)

Recently, a petition has been submitted by some activists to the US Food and Drug Administration (FDA) to reclassify toxic 3 Os (Oxybenzone, Octinoxate and Octocrylene) used in sunscreen.

They demanded that chemicals be shifted to “Not Generally Recognized As Safe & Effective” (GRASE Category II).

Page 56: DAILY Q & A SERIES

DAILY Q & A SERIES

For Enrolment Join Telegram Channel: SIMPLIFIED_GS

Toxic 3 Os:

o Oxybenzone (OBZ), avobenzone (AVB), octocrylene (OCR) and octinoxate (OMC) are UltraViolet (UV) filters commonly added to chemical sunscreens.

o These UV filters are known to widely contaminate the environment through a variety of anthropogenic sources, including sewage discharge.

Related Concerns:

o The 3Os are active ingredients present in more than two-thirds of all sunscreens and they pose a threat to public health, marine life and coral reefs.

o Octocrylene in sun protection products degrades into benzophenone.

Benzophenone is a carcinogen that can also interfere with key hormones and reproductive organs.

o They cause human cell damage that has been linked to cancer, disrupt hormones, have been found in breast milk, blood and urine and are known allergens.

o Oxybenzone is particularly toxic to corals at concentrations as low as a few parts per trillion.

Q87) Kaushalacharya Awards 2021 is an initiative of:

a) Union Ministry of Skill Development & Entrepreneurship

b) Union Ministry of Finance

c) NITI Aayog

d) Ministry of Women and Child development

Ans : a)

Union Education Minister and Skill Development Minister Dharmendra Pradhan virtually presented Kaushalacharya Awards 2021 to 41 skill trainers for their exemplary contribution to the skill ecosystem.

These 41 trainers are from several initiatives and training programmes of Skill India— Directorate General of Training (DGT), Apprenticeship, Pradhan Mantri Kaushal Vikas Yojana (PMKVY), Jan Shikshan Sansthan (JSS) and Entrepreneurship.

Industrial Training Institutes (ITIs), pillars of India’s vocational training ecosystem, had as many as 11 trainers who were felicitated. Kaushalacharya Awards is an initiative of the Ministry of Skill Development & Entrepreneurship (MSDE). The Kaushalacharya Awards is an annual celebration that focuses on honoring the skill trainers and the impact created by them on India’s youth.

Page 57: DAILY Q & A SERIES

DAILY Q & A SERIES

For Enrolment Join Telegram Channel: SIMPLIFIED_GS

Q88) Sequoia National Park, often mentioned in news, is located in:

a) France

b) Germany

c) U.K

d) U.S.A

Ans : d)

Two wildfires in California – one called the Colony fire and the other called Paradise – are burning through the Sequoia National Park in the Sierra Nevada that is home to some of the largest trees in the world.

Among these trees is the world’s largest tree popularly known as General Sherman, which firefighters are now trying to protect from the blaze. Q89) India's Shefali Juneja has been elected as first woman chairperson of which UN agency's security committee?

a) ICAO

b) UNTACD

c) UNHCR

d) UNICEF

Ans : a)

India's Shefali Juneja has been elected as the first woman Chairperson of the International Civil Aviation Organization (ICAO) Aviation Security Committee. India has taken up the responsibility of the ICAO Aviation Security Committee after a gap of 12 years.

Q90) Consider the correct statement about bad bank:

1) A bad bank is a corporate structure that isolates risky assets held by banks in a

separate entity.

2) The bad bank or NARCL will pay up to 15 per cent of the agreed value for the loans in

cash and the remaining 85 per cent would be government-guaranteed security

receipts.

Page 58: DAILY Q & A SERIES

DAILY Q & A SERIES

For Enrolment Join Telegram Channel: SIMPLIFIED_GS

a) 1 only

b) 2 only

c) Both 1 and 2

d) Neither 1 nor 2

Ans: c)

The Union Cabinet on Monday approved Rs 30,600 crore government guarantee for the National Asset Reconstruction Company (NARCL), thereby paving the way for operationalisation of bad bank. One of the key ideas behind formation of bad banks is to de-stress the balance sheets of the banks.

A bad bank is a corporate structure that isolates risky assets held by banks in a separate entity. It is established to buy non-performing assets (NPAs) from a bank at a price that is determined by the

bad bank itself. The bad bank or NARCL will pay up to 15 per cent of the agreed value for the loans in cash and the

remaining 85 per cent would be government-guaranteed security receipts. The government guarantee would be invoked if there is loss against the threshold value. The value of bad loans being carved out of bank books for transfer to the NARCL is around Rs 2 lakh

crore. About Rs 90,000 crore in bad loans will be transferred in the first phase. The guarantee of Rs 30,600 crore will cover the entire pool of Rs 2 lakh crore.

Q91) Consider the correct statement about Sea cucumbers : 1) are marine invertebrates that live on the seafloor found generally in tropical regions. 2) Recently, the Indian Coast Guard (ICG) has seized two tonnes of sea cucumber, a banned

marine species, in the Gulf of Mannar . a) 1 only b) 2 only c) Both 1 and 2 d) Neither 1 nor 2

Ans : (c)

Recently, the Indian Coast Guard (ICG) has seized two tonnes of sea cucumber, a banned marine species, in the Gulf of Mannar and Palk Bay areas in Tamil Nadu.

o Sea cucumbers are marine invertebrates that live on the seafloor found generally in tropical regions. They're named for their unusual oblong shape that resembles a fat cucumber.

o There are about 1,250 species of sea cucumber, all of which belong to the taxonomic class Holothuroidea.

This class falls under the Echinodermata phylum, which also includes many other well-known marine invertebrates, such as sea stars, sea urchins and sand dollars.

o They are crucial to maintain the balance of ocean habitats.

Q92 ) Consider the correct statement about Delhi-Mumbai Expressway:

1) It will be the world’s longest highway once completed

Page 59: DAILY Q & A SERIES

DAILY Q & A SERIES

For Enrolment Join Telegram Channel: SIMPLIFIED_GS

2) It will start from Delhi and will end around Mira Bhayander near Mumbai. a) 1 only b) 2 only c) Both 1 and 2 d) Neither 1 nor 2

Ans: a)

Recently, the Union Minister for Road Transport and Highways concluded the two-day review of the work progress on the eight-lane Delhi-Mumbai Expressway.

It will be the world’s longest highway once completed. It is expected to be completed by March 2023.

o It is a 1380-km eight-lane expressway that can be expanded to a 12-lane expressway depending on the volume of traffic.

Page 60: DAILY Q & A SERIES

DAILY Q & A SERIES

For Enrolment Join Telegram Channel: SIMPLIFIED_GS

o It will start from Haryana's Sohna and will end around Mira Bhayander near Mumbai.

Q93 ) Which ministry has launched a Pan-India Special Campaign called “Ek Pahal Drive?”

a) Ministry of Law and Justice b) Ministry of Rural Development c) Ministry of Corporate Affairs d) Ministry of Finance

Ans : a)

A Pan-India Special Campaign called “Ek Pahal Drive” for Justice Delivery at doorstep was launched by the Ministry of Law and Justice. This campaign was launched in a bid to encourage mass registration under Tele-Law. Department of Justice and NALSA launched this campaign. The purpose of this campaign is to realize the desire of Indian citizens to have access to justice.

Q94) Consider the INCORRECT statement about Adaptive Refresh Rate:

1) Refresh rate means how many times the image on a screen can be updated or

“refreshed” every second – much faster than your eye can detect.

2) A static website will be played at a 10Hz refresh rate and a game at a 120Hz

refresh rate.

a) 1 only

b) 2 only

c) Both 1 and 2

d) Neither 1 nor 2

Ans : d)

The recent iPhone 13 Pro range is offering a 120Hz high-refresh-rate display which Apple is calling ProMotion.

Refresh Rate: Whether it is a TV, a PC monitor, or a smartphone, all devices constantly change the pixels they display

to project the appearance of motion. Simply put, refresh rate means how many times the image on a screen can be updated or “refreshed”

every second – much faster than your eye can detect. A 60Hz TV can refresh the screen 60 times per second, while a 120Hz on the iPhone 13 Pro can refresh

the screen 120 times per second. The higher the refresh rate, the more times per second the screen is refreshing its pixels. Adaptive Refresh Rate:

When a display has an adaptive refresh rate the refresh rate changes from 10Hz to 120Hz depending on the content being played — some Samsung phones offer between 48Hz and 120Hz.

So, a static website will be played at a 10Hz refresh rate and a game at a 120Hz refresh rate. This means there is little chance of under delivering or over delivering refresh rate.

Page 61: DAILY Q & A SERIES

DAILY Q & A SERIES

For Enrolment Join Telegram Channel: SIMPLIFIED_GS

Q95) Which of the following nations own and operate Nuclear-powered submarines currently?

1. Germany

2. India

3. U.S.A

Select the correct answer using the code given below:

a) 3 only

b) 1 and 2 only

c) 2 and 3 only

d) 1, 2 and 3

Ans: c)

The US and UK are set to provide Australia with the technology to deploy nuclear-powered submarines, as part of the first initiative under the new trilateral security partnership AUKUS.

A nuclear-powered submarine, as the name suggests, is powered by a nuclear reactor but it is not a nuclear weapon.

Every nuclear-powered submarine draws from its own miniature nuclear reactor onboard, which is typically fuelled with uranium.

For such a reactor to work, uranium has to be ‘enriched’ to contain 50 per cent of a key isotope, uranium-235.

Natural uranium consists of approximately 99.3 per cent of the isotope uranium-238 and only 0.7 per cent of uranium-235. The process of enrichment can be carried out through gaseous diffusion, gas centrifuges or laser isotope separation.

Nuclear-powered submarines do not have the same limitations that face conventional submarines on weapons storage, speed and endurance. They can stay completely submerged for many months, limiting the opportunities for detection by adversaries.

Only six nations own and operate these submarines currently: China, France, India, Russia, the UK and the US Q96) The Sakarya is one of the longest river in:

a) France

b) U.K

Page 62: DAILY Q & A SERIES

DAILY Q & A SERIES

For Enrolment Join Telegram Channel: SIMPLIFIED_GS

c) Germany

d) Turkey

Ans : d)

A couple of hours before Capt Amarinder Singh tendered his resignation as chief minister of Punjab, former Punjab Congress chief Sunil Jakhar made a tweet: “Kudos to Shri Rahul Gandhi for adopting Alexandrian solution to this Punjabi version of Gordian knot".

Often used as a metaphor, a ‘Gordian knot’ refers to a very complex problem, which is apparently nearly impossible to resolve.

The origin of the phrase is traced to the ancient Greece, 333 BC, and is ascribed to Midas, the son of King Gordius of Phrygia, a kingdom in the west central part of Anatolia — the present day Asian Turkey — and to the south of Black Sea.

Phrygia was centred on the Sangarios River. The Sakarya is the third longest river in Turkey.

Q97) Consider the correct statement about Humboldt penguins:

1) Humboldt penguins are endemic to the Pacific coasts of Chile and Peru. 2) They are classified as Endangered by the IUCN Red List.

a) 1 only b) 2 only c) Both 1 and 2 d) Neither 1 nor 2

Ans : a)

Humboldt penguins:

Last week, Mumbai’s Byculla Zoo announced the addition of two new Humboldt penguin chicks this year. The two join seven adult Humboldt penguins at Byculla Zoo.

Humboldt penguins are a medium-sized species among at least 17 species. Humboldt penguins have an average height of just over 2 ft (Among the Species, the

Emperor penguin is the largest and stands at over 4 ft tall). The Humboldt penguin (Spheniscus humboldti) belongs to a genus that is commonly

known as the ‘banded’ group (Penguins are divided into six genera). Humboldt penguins are endemic to the Pacific coasts of Chile and Peru. They are so named because their habitat is located near the Humboldt Current, a large

oceanic upwelling characterised by cold waters. They are classified as vulnerable by the IUCN Red List.

Q98) stablecoin recently seen in the news, is associated with

Page 63: DAILY Q & A SERIES

DAILY Q & A SERIES

For Enrolment Join Telegram Channel: SIMPLIFIED_GS

a) COVID-19

b) a type of cryptocurrency

c) a book

d) super computer

Ans : b)

A stablecoin is a type of cryptocurrency that is typically pegged to an existing government -backed currency.

The majority of the dozens of stablecoins that currently exist use the dollar as their benchmark asset, but many are also pegged to other fiat currencies issued by governments like the euro and yen.

As a result, the price of stablecoins fluctuates very little, unlike high -profile cryptocurrencies like bitcoin and ethereum that are prone to sudden ups and downs.

Stablecoins are useful because they allow people to transact more seamlessly in cryptocurrencies that function as investments, such as Bitcoin. They form a bridge between old-world money and new-world crypto.

The first stablecoin, created in 2014, was Tether, which many other stablecoins are modeled after

Q99) Consider the INCORRECT statement about General Sherman:

1) General Sherman tree is the world’s largest in terms of volume and exists in the Giant Forest sequoia grove of the national park.

2) It is located in the U.S. state of California.

a) 1 only

b) 2 only

c) Both 1 and 2

d) Neither 1 nor 2

Ans : d)

General Sherman:

The General Sherman tree is the world’s largest in terms of volume and exists in the Giant Forest sequoia grove of the national park. As per recent estimates, General Sherman is about 2,200 years old.

The tree stands at a height of 275 feet (taller than the leaning tower of Pisa) and has a diameter of 36 feet at the base.

It is located in the U.S. state of California.

The tree is threatened by California wildfires.

Page 64: DAILY Q & A SERIES

DAILY Q & A SERIES

For Enrolment Join Telegram Channel: SIMPLIFIED_GS

Q100) With which organization, NITI Aayog has partnered to provide free access to education to

students in 112 Aspirational Districts in India?

a) Unacademy b) BYJU c) Tata Institute of Fundamental Research d) IIT Kanpur

Ans: b)

NITI Aayog has tied up with BYJU`S to provide free education opportunities for students in 112

Aspirational Districts in India. As part of the partnership, Byju’s will provide students with high-quality

and technology-oriented learning programs free of charge to ensure high-quality education. This project

will provide for the establishment of a dedicated working group, which in turn should help establish a

system of knowledge, innovation and strategic support.

Q101) Consider the correct statement about Global Innovation Index (GII) 2021 : 1) India has climbed two spots and has been ranked 56th in the GII 2021 rankings. 2) GII is launched by the World Intellectual Property Organization . 3) Republic of Korea joins the top 5 of the GII for the first time in 2021.

a) 1 only b) 1 and 2 c) 2 and 3 d) 1,2 and 3

Ans : (c)

Recently, India has climbed two spots and has been ranked 46th in the Global Innovation Index (GII) 2021 rankings.

Global Innovation Index (GII) 2021:

Launch: The GII is launched by the World Intellectual Property Organization (WIPO), a specialized agency of the United Nations.

The GII aims to capture the multi-dimensional facets of innovation ranking and rich analysis referencing around 132 economies.

Partnership: It is published in partnership with the Portulans Institute and other corporate partners:

The Brazilian National Confederation of Industry (CNI), Confederation of Indian Industry (CII), Ecopetrol (Colombia) and the Turkish Exporters Assembly (TIM).

Page 65: DAILY Q & A SERIES

DAILY Q & A SERIES

For Enrolment Join Telegram Channel: SIMPLIFIED_GS

Indicators: The index ranks world economies according to their innovation capabilities and consists of roughly 80 indicators grouped into innovation inputs and outputs.

Innovation inputs: Institutions; Human capital and research; Infrastructure; Market sophistication; Business sophistication.

Innovation outputs: Knowledge and technology outputs; Creative outputs

Global Performance:

Top Five: Switzerland, Sweden, U.S., and U.K. continue to lead the innovation ranking, and have all ranked in the top 5 in the past three years.

The Republic of Korea joins the top 5 of the GII for the first time in 2021.

Asian Countries: Four Asian economies feature in the top 15: Singapore (8), China (12), Japan (13) and Hong Kong, China (14).

India’s Performance:

India has been on an upward trajectory over the past few years in the GII.

India has shot up from a rank of 81 in 2015 to 46 in 2021.

India performs better in innovation outputs than innovation inputs in 2021.

This year India ranks 57th in innovation inputs, the same as last year but higher than 2019.

As for innovation outputs, India ranks 45th. This position is the same as last year but higher than 2019.

India ranks 2nd among the 34 lower middle-income group economies.

India ranks 1st among the 10 economies in Central and Southern Asia.

The government attributed the country’s improved performance to the departments of atomic energy, science and technology, biotechnology and space.

Q102 ) Which of the following state/UTs has decided to implement the Forest Rights Act (FRA),

2006, which will elevate the socio-economic status of a sizable section of the 14-lakh-strong

population of tribals and nomadic communities.

a) Jammu and Kashmir

b) Sikkim

c) Odisha

d) Goa

Page 66: DAILY Q & A SERIES

DAILY Q & A SERIES

For Enrolment Join Telegram Channel: SIMPLIFIED_GS

Ans: a)

Recently, the Jammu and Kashmir government has decided to implement the Forest Rights Act (FRA), 2006, which will elevate the socio-economic status of a sizable section of the 14-lakh-strong population of tribals and nomadic communities.

o FRA enacted in 2006 recognises the rights of forest-dwelling tribal communities and other traditional forest dwellers to forest resources on which these communities were dependent for a variety of needs, including livelihood, habitation and other sociocultural needs.

o It recognizes and vest the forest rights and occupation in Forest land in Forest Dwelling Scheduled Tribes (FDST) and Other Traditional Forest Dwellers (OTFD) who have been residing in such forests for generations.

o It strengthens the conservation regime of the forests while ensuring livelihood and food security of the FDST and OTFD.

o The Gram Sabha is the authority to initiate the process for determining the nature and extent of Individual Forest Rights (IFR) or Community Forest Rights (CFR) or both that may be given to FDST and OTFD.

Q103 ) Consider the INCORRECT statement about Vishnuonyx:

1) Fossils of Vishnuonyx now extinct otters were first discovered in sediments found in the foothills of the Himalayas.

2) The species was discovered from a 11.4-million-year-old strata in the area of Hammerschmiede, which is a fossil site in Bavaria, Germany.

a) 1 only

b) 2 only

c) Both 1 and 2

d) Neither 1 nor 2

Ans : d)

Between 12.5 million and 14 million years ago, members of a genus of otters called Vishnuonyx lived in the major rivers of southern Asia.

Fossils of these now extinct otters were first discovered in sediments found in the foothills of the Himalayas. Now, a newly found fossil indicates it had travelled as far as Germany. The discovery has been described in the Journal of Vertebrate Paleontology.

Researchers from the Universities of Tübingen and Zaragoza have discovered the fossil of a previously unknown species, which they have named Vishnuonyx neptuni, meaning ‘Neptune’s Vishnu’.

The species was discovered from a 11.4-million-year-old strata in the area of Hammerschmiede, which is a fossil site in Bavaria, Germany that has been studied for about 50 years.

This is the first discovery of any member of the Vishnuonyx genus in Europe; it is also its most northern and western record till date.

Page 67: DAILY Q & A SERIES

DAILY Q & A SERIES

For Enrolment Join Telegram Channel: SIMPLIFIED_GS

Vishnuonyx were mid-sized predators that weighed, on average, 10-15 kg. Before this, the genus was known only in Asia and Africa (recent findings show that Vishnuonyx reached East Africa about 12 million years ago.

Q104)Consider the correct statement about State Food Safety Index :

1) It is an initiative of NITI Aayog.

2) This year, among the larger states, Gujarat was the top ranking state, followed by Kerala and Tamil Nadu.

a) 1 only

b) 2 only

c) Both 1 and 2

d) Neither 1 nor 2

Ans : b)

Union Minister for Health and Family Welfare Mansukh Mandaviya released Food Safety and Standards Authority of India (FSSAI)’s 3rd State Food Safety Index (SFSI) to measure the performance of States across five parameters of food safety.

The Union Minister for Health and Family Welfare felicitated nine leading States/UTs based on the ranking for the year 2020-21 for their impressive performance.

This year, among the larger states, Gujarat was the top ranking state, followed by Kerala and Tamil Nadu.

Among the smaller states, Goa stood first followed by Meghalaya and Manipur. Among UTs, Jammu & Kashmir, Andaman & Nicobar Islands and New Delhi secured top ranks. He also flagged off 19 Mobile Food Testing Vans (Food Safety on Wheels) to supplement the food safety

ecosystem across the country taking the total number of such mobile testing vans to 109. Trans Fat

The Union Minister released the results of PAN-India survey for identifying the presence of industrially produced trans fatty acid content in the selected foods.

Samples of various packaged food items under six pre-defined food categories were collected from 419 cities/districts across 34 States/UTs.

Overall, only 84 samples, i.e. 1.34%, have more than 3% industrially produced trans fats from the total of 6245 samples; India is on the right track of becoming industrial trans fats free in the 75th year of India’s independence.

Q105) Consider the correct statement about 5G technology:

1) 5G is the next generation of mobile broadband that will eventually replace, or at

least augment 4G LTE connection.

2) Increased capacity on 5G networks can minimize the impact of load spikes.

a) 1 only

b) 2 only

c) Both 1 and 2

Page 68: DAILY Q & A SERIES

DAILY Q & A SERIES

For Enrolment Join Telegram Channel: SIMPLIFIED_GS

d) Neither 1 nor 2

Ans: c)

Vodafone Idea (Vi) claimed to have achieved a peak 5G data speed of 3.7Gbps on the mmWave spectrum band in a recent test conducted in Pune, Maharashtra. Peak download speeds of up to 1.5Gbps in the 3.5Ghz band 5G trial network in Gandhinagar and Pune.

5G is the next generation of mobile broadband that will eventually replace, or at least augment 4G LTE connection.

Features and benefits of the 5G technology:

1. Operate in the millimeter wave spectrum (30-300 GHz) which have the advantage of sending large amounts of data at very high speeds.

2. Operate in 3 bands, namely low, mid and high frequency spectrum. 3. Reduced latency will support new applications that leverage the power of 5G, the Internet

of Things (IoT), and artificial intelligence. 4. Increased capacity on 5G networks can minimize the impact of load spikes, like those that

take place during sporting events and news events.

Q106) Recently, ‘Sea snot’ outbreak in ?

a) Iran

b) Turkey

c) Albania

d) Japan

Ans : b)

The sea snot that dominated the landlocked Marmara Sea earlier this year is nowhere to be seen on the surface nowadays, but its fallout appears to be bigger than experts initially hoped.

Impact of sea snot:

1. Overall, 60% of species have already disappeared. 2. The layers have sunk and are beginning to decompose. 3. The decomposition consumes oxygen in the water, which in turn promotes the formation

of new marine mucilage. 4. In October, the conditions will be particularly favorable for a new spread. Therefore, the

sludge may be visible on the surface again in November. 5. The slime could also spread to the Black Sea and the Aegean and may cause a regional

ecological crisis.

Turkey’s Sea of Marmara, that connects the Black Sea to the Aegean Sea, has witnessed the largest outbreak of ‘sea snot’. The sludge has also been spotted in the adjoining Black and Aegean seas.

What is sea snot?

Page 69: DAILY Q & A SERIES

DAILY Q & A SERIES

For Enrolment Join Telegram Channel: SIMPLIFIED_GS

It is a slimy layer of grey or green sludge, which can cause considerable damage to the marine ecosystem.

It is formed when algae are overloaded with nutrients. A ‘sea snot’ outbreak was first recorded in the country in 2007. Back then, it was also

spotted in the Aegean Sea near Greece.

Overloading of nutrients happens because of warm weather caused by global warming, water pollution, uncontrolled dumping of household and industrial waste into the seas etc.

Q107) Which of the following countries comprises the “Horn of Africa”, often seen in the news?

1. Eritrea

2. Kenya

3. Ethiopia

4. Somalia

Choose the correct answers using the codes given below:

(a) 1, 2 and 3 only

(b) 2, 3 and 4 only

(c) 1, 3 and 4 only

(d) All of the above

Ans : c)

Recently, the observers had feared that military operation in the Tigray region of Ethiopia would have ramifications in the Horn of Africa region at large. Option C is correct: The North-Eastern region of the African continent including the countries of Sudan, Eritrea, Ethiopia, Djibouti and Somalia is known as the Horn of Africa since the group of countries form a horn-like landmass over the peninsula.

Q108) ‘Anakkayam Hydel Project’ was often seen in the news, it is proposed by which of the following

states?

(a) Kerala

(b) Tamil Nadu

(c) Karnataka

(d) Maharashtra

Page 70: DAILY Q & A SERIES

DAILY Q & A SERIES

For Enrolment Join Telegram Channel: SIMPLIFIED_GS

Ans : a)

Recently, Environmentalists and tribal communities have come together to protest against the

Anakkayam Small Hydro Electric Project in Kerala. The Kerala government approved the Athirapally

Hydro Electric Project (AHEP) on the Chalakudy river in Thrissur district of Kerala. Chalakudy River

originates in the Anamalai region of Tamil Nadu and is joined by its major tributaries Parambikulam,

Kuriyarkutti, Sholayar, Karapara and Anakayam in Kerala. It flows through Palakkad, Thrissur and

Ernakulam districts of Kerala.

Q109) Recently “Tristan da Cunha” was in the news. Which of the following statement/(s) are correct in this regard?

1. It is a group of volcanic islands.

2. It is an overseas territory of the UK located in the South Atlantic Ocean.

(a) 1 only

(b) 2 only

(c) Both 1 and 2

(d) Neither 1 nor 2

Ans : c)

Tristan da Cunha is a remote group of volcanic islands in the south Atlantic Ocean. It is a British Overseas Territory located in the south Atlantic Ocean.

Q110) Which one of the following is not an instrument of selective credit control in India?

(a) Regulation of consumer credit

(b) Rationing of credit

(c) Margin requirements

(d) Variable cash reserve ratios

Ans: d)

Variable Reserve Ratio (Cash Reserve Ratio) is aimed to control only the volume of credit (quantitative

method) not both volume and purpose of credit for which banks give loans. Qualitative method and

the selective control method are used for purposes. It has a number of limitations.

22 September 2021

Q1) Which state has inaugurated South Asia’s largest Product development centre ‘Digital Hub’, to

support start-ups?

Page 71: DAILY Q & A SERIES

DAILY Q & A SERIES

For Enrolment Join Telegram Channel: SIMPLIFIED_GS

a) Kerala b) Karnataka c) Uttarakhand d) Uttar Pradesh

Ans : (a)

Chief Minister of Kerala Pinarayi Vijayan inaugurated Digital Hub with a capacity to support 200 start-ups via video conferencing recently. The product development centre, one of the largest in South Asia, at the Technology Innovation Zone (TIZ) in Kalamassery, Kochi. It aims to provide direct employment opportunities for 2500 people at the Hub. The Kerala Startup Mission (KSUM) leads the implementation of the project.

Q2 ) Consider the correct statement about Aroma Mission:

1) To promote the cultivation of aromatic crops for essential oils that are in great demand by the aroma industry.

2) The nodal laboratory is CSIR-Central Institute of Medicinal and Aromatic Plants (CSIR-CIMAP), Lucknow.

a) 1 only

b) 2 only

c) Both 1 and 2

d) Neither 1 nor 2

Ans: c)

Recently, the Union Minister of State Science & Technology has proposed Integrated Aroma Dairy Entrepreneurship for Jammu & Kashmir to augment the income of farmers.

The Aroma Mission, also popularly referred as "Lavender or Purple Revolution", has started from J&K and transformed the lives of farmers who are able to grow lavender, make lucrative profit and improve their lives.

Earlier, the floriculture mission was launched in 21 States and Union Territories.

Aroma Mission:

o Objectives:

To promote the cultivation of aromatic crops for essential oils that are in great demand by the aroma industry.

To enable Indian farmers and the aroma industry to become global leaders in the production and export of some other essential oils on the pattern of menthol mint.

Page 72: DAILY Q & A SERIES

DAILY Q & A SERIES

For Enrolment Join Telegram Channel: SIMPLIFIED_GS

To provide substantial benefits to the farmers in achieving higher profits, utilization of waste lands and protection of their crops from wild and grazing animals.

o Nodal Agencies:

The nodal laboratory is CSIR-Central Institute of Medicinal and Aromatic Plants (CSIR-CIMAP), Lucknow.

The participating laboratories are CSIR-Institute of Himalayan Bioresource Technology (CSIR-IHBT), Palampur; CSIR-Indian Institute of Integrative Medicine (CSIR-IIIM), Jammu etc.

o Coverage:

The scientific interventions made under the mission project provided assured benefits to the growers of Vidarbha, Bundelkhand, Gujarat, Marathwada, Rajasthan, Andhra Pradesh, Odisha and other states where farmers are exposed to frequent episodes of weather extremes and account for maximum suicides.

Aromatic Plants include lavender, damask rose, mushk bala, etc.

Q3 ) Consider the incorrect statement about GST Council:

1) It is a constitutional body (Article 269A).

2) GST Council is chaired by the Secretary of Finance Minister.

a) 1 only

b) 2 only

c) Both 1 and 2

d) Neither 1 nor 2

Ans : c)

Recently, the 45th Goods and Services Tax (GST) Council meeting was held.

Page 73: DAILY Q & A SERIES

DAILY Q & A SERIES

For Enrolment Join Telegram Channel: SIMPLIFIED_GS

Extension of Concessional GST Rates:

o The Council decided to extend the GST relief on several drugs related to Covid-19 treatment till December 2021.

Food Delivery Apps to Collect GST:

o Online food delivery aggregator firms such as Swiggy and Zomato will now be liable to pay GST and not the restaurant partners.

Currently, online bills generated by food aggregators already have a tax component in it.

The taxed amount is paid back to the restaurant partners who are then expected to pay this amount to the government.

Petrol and Diesel will not come under GST Regime:

o The council has decided not to bring petrol and diesel under the GST regime. States vehemently opposed the inclusion of the fuels while raising concerns on revenue buoyancy during the meet.

If petrol and diesel come under the GST regime, prices will become mostly uniform across all states as the different excise and VAT rates that the Centre and the states impose would then be done away with.

This would help bring down diesel and petrol prices greatly, which has touched new highs in the recent past.

GST on Fortified Rice Reduced:

Page 74: DAILY Q & A SERIES

DAILY Q & A SERIES

For Enrolment Join Telegram Channel: SIMPLIFIED_GS

o The GST rate on fortified rice kernels for schemes like integrated child development schemes has been recommended to be reduced from 18% to 5%.

GoM to Look After Rate Rationalisation:

o A Group of state ministers (GoM) will be established to look after the rate rationalisation related issues to correct the inverted duty structure and to take steps to augment revenues.

An inverted duty structure arises when the taxes on output or final product is lower than the taxes on inputs, creating an inverse accumulation of input tax credit which in most cases has to be refunded.

Inverted duty structure has implied a stream of revenue outflow for the government prompting the government to relook the duty structure.

o Other GoMs will be set up in order to look after issues of e-way bills, FASTAGs, compliances, technology, plugging of loopholes, composition schemes etc.

GST Council

It is a constitutional body (Article 279A) for making recommendations to the Union and State

Government on issues related to Goods and Services Tax.

The GST Council is chaired by the Union Finance Minister and other members are the Union

State Minister of Revenue or Finance and Ministers in-charge of Finance or Taxation of all the

States.

It is considered as a federal body where both the centre and the states get due representation.

Q4) Correctly matched pair of IUCN Red List Status :

1) White Rhino : Near Threatened.

2) Black Rhino : Critically endangered.

3) Greater One Horned : Vulnerable.

4) Javan : Critically Endangered

5) Sumatran Rhino : Critically Endangered

a) 1 ,2 and 3

b) 2,3 and 4

c) 4 and 5

d) 1,2,3,4 and 5

Ans : d)

World Rhino Day is observed on 22th September to spread awareness for all five species of rhino and work being done to save them.

Page 75: DAILY Q & A SERIES

DAILY Q & A SERIES

For Enrolment Join Telegram Channel: SIMPLIFIED_GS

It was first announced by the World Wildlife Fund (WWF) - South Africa in 2010. The species of rhinoceros are on the verge of extinction due to persistent poaching and habitat loss over several decades.

There are five species of rhino – white and black rhinos in Africa, and the greater one-horned, Javan and Sumatran rhino species in Asia.

o IUCN Red List Status:

White Rhino: Near Threatened.

Black Rhino: Critically endangered.

Greater One Horned: Vulnerable.

Javan: Critically Endangered

Sumatran Rhino: Critically Endangered

Theme 2021: Keep the five Alive.

Objectives: Strengthening protection, Expanding the distribution range, Research and monitoring, Adequate and sustained funding.

Q5) Which organisation/Institute has developed a super-hydrophobic cotton composite with Metal-

Organic Framework (MOF) that can clean-up marine oil-spill.

a) CSIR Lucknow

b) IIT, Guwahati

c) ISRO

d) IISc Bangaluru

Ans: b)

Recently, the Indian Institute of Technology (IIT), Guwahati, has developed a super-hydrophobic cotton composite with Metal-Organic Framework (MOF) that can clean-up marine oil-spill.

Earlier, a study confirmed that Stimulating Bacteria (Bioremediation) with nutrients in the cold seawaters of the Canadian Arctic can help decompose diesel and Other Petroleum Oil after Oil Spills.

It is a novel MOF composite, which is a highly porous and water-repellent material and can absorb oil selectively from an oil-water mixture.

MOF are crystalline porous solids composed of a three-dimensional (3D) network of metal ions held in place by multidentate organic molecules suitable for solid-phase extraction.

Q6) Recently, the Cumbre Vieja volcano in La Palma erupted in

Page 76: DAILY Q & A SERIES

DAILY Q & A SERIES

For Enrolment Join Telegram Channel: SIMPLIFIED_GS

a) Atlantic ocean

b) Pacific ocean

c) Indian Ocean

d) Arctic Ocean

Ans : a)

Recently, the Cumbre Vieja volcano in La Palma, one of Spain's Canary Islands, erupted.

o The Cumbre Vieja volcano erupted for the first time in 50 years. The last time hot molten lava erupted and spread was in 1971.

o La Palma is the most north-westerly island of the Canary Islands, Spain.

It’s geography is a result of a volcanic formation and has an area of 708 square kilometres making it the fifth largest of the eight main Canary Islands.

Its highest mountain is the Roque de los Muchachos, at 2,423 metres being second among the peaks of the Canaries only to the peaks of the Teide massif on Tenerife.

o The Canary Islands are a group of ocean island volcanoes located off the coast of western Africa that have been formed by volcanic activity since around 20 million years ago. It comprises the Spanish provinces of Las Palmas and Santa Cruz de Tenerife.

Other Recently Erupted Volcanoes:

o Kilauea Volcano: Hawaii.

o Fukutoku-Okanoba Volcano: Japan

Page 77: DAILY Q & A SERIES

DAILY Q & A SERIES

For Enrolment Join Telegram Channel: SIMPLIFIED_GS

o Sangay Volcano: Ecuador

o Taal Volcano: Philippines

o Mt. Sinabung, Merapi volcano, Semeru volcano (Indonesia)

Volcanoes in India:

o Barren Island, Andaman Islands (India's only active volcano)

o Narcondam, Andaman Islands

o Baratang, Andaman Islands

o Deccan Traps, Maharashtra

o Dhinodhar Hills, Gujarat

o Dhosi Hill, Haryana

Q7) Which of the following beaches has accorded the Blue Flag Certification?

1. Ghoghla-Diu

2. Kovalam

3. Eden

Select the correct answer using the code given below:

a) 1 only

b) 1 and 2 only

c) 2 and 3 only

d) 1, 2 and 3

Ans : d)

The globally recognized and the coveted International eco-label "Blue Flag”, has accorded the Blue Flag Certification for 2 new beaches this year –Kovalam in Tamil Nadu and Eden in Puducherry beaches.

Foundation for Environment Education in Denmark (FEE) which accords the globally recognized eco-label - Blue Flag certification, has also given re-certification for 8 nominated beaches Shivrajpur-Gujarat, Ghoghla-Diu, Kasarkod and Padubidri-Karnataka, Kappad-Kerala, Rushikonda- Andhra Pradesh, Golden-Odisha and Radhanagar- Andaman and Nicobar, which were awarded the Blue Flag certificate last year.

Page 78: DAILY Q & A SERIES

DAILY Q & A SERIES

For Enrolment Join Telegram Channel: SIMPLIFIED_GS

The Ministry of Environment embarked upon a highly acclaimed & flagship program Beach Environment & Aesthetics Management Services (BEAMS).

BEAMS is one of the initiatives under ICZM approach that the MoEF&CC has undertaken for the sustainable development of coastal regions of India, with a prime objective to protect and conserve the pristine coastal and marine ecosystems through holistic management of the resources.

This was aimed for achieving the globally recognized and the coveted International eco-label "Blue Flag” , accorded by International Jury comprising of members from IUCN, UNWTO ,UNEP, UNESCO etc.

FEE Denmark conduct regular monitoring & audits for strict compliance of the 33 criteria at all times. A waving “Blue Flag” is an indication of 100% compliance to these 33 stringent criteria and sound health of the beach.

The objective of BEAMS program is to abate pollution in coastal waters, promote sustainable development of beach facilities, protect & conserve coastal ecosystems & natural resources, and seriously challenge local authorities & stakeholders to strive and maintain high standards of cleanliness, hygiene & safety for beachgoers.

Q8)Recently, Archeologists have found ‘shankhalipi’ inscriptions on the stairs on an ancient temple

dating back to the Gupta period in

a) Bihar

b) Madhya Pradesh

c) Uttar Pradesh

d) Assam

Ans : c)

Archeologists have found ‘shankhalipi’ inscriptions on the stairs on an ancient temple dating back to the Gupta period in Uttar Pradesh’s Etah district.

The inscriptions mention ‘Sri Mahendraditya’, the title of Kumaragupta I of the Gupta dynasty.

Significance of the latest findings:

Two decorative pillars close to one another, with human figurines have been discovered at Bilsarh site. The discovery becomes significant since only two other structural temples from the Gupta age have been found so far — Dashavatara Temple (Deogarh) and Bhitargaon Temple (Kanpur Dehat).

Q9)Consider the correct statement about Plant Discoveries 2020

1) There are 14 new macro and 31 new micro fungi species recorded from various parts of India.

2) 22% of the discoveries were made from the Western Ghats followed by the Western Himalayas (15%), the Eastern Himalayas (14%) and the Northeast ranges (12%).

Page 79: DAILY Q & A SERIES

DAILY Q & A SERIES

For Enrolment Join Telegram Channel: SIMPLIFIED_GS

a) 1 only

b) 2 only

c) Both 1 and 2

d) Neither 1 nor 2

Ans : c)

Plant Discoveries 2020:

The Botanical Survey of India, in its new publication Plant Discoveries 2020 has added 267 new taxa/ species to the country’s flora.

The 267 new discoveries include 119 angiosperms; 3 pteridophytes; 5 bryophytes, 44 lichens; 57 fungi, 21 algae and 18 microbes.

There are 14 new macro and 31 new micro fungi species recorded from various parts of India.

22% of the discoveries were made from the Western Ghats followed by the Western Himalayas (15%), the Eastern Himalayas (14%) and the Northeast ranges (12%).

Q10) Which two countries are set to participate in the Defence Exercise ‘Samudra Shakti’?

a) India and Indonesia b) India and Nepal c) India and China d) India and Bhutan

Ans: a)

Indian & Indonesian Navy are participating in the two-day bilateral maritime exercise ‘Samudra Shakti’ in Jakarta scheduled from September 20-22. The exercise aims to strengthen the bilateral relationship, enhance mutual understanding and interoperability in maritime operations between the two navies. The Indian Navy ships Shivalik and Kadmatt, and Anti-Submarine Warfare-Long Range Maritime Reconnaissance Aircraft P8I are participating in the exercise.

23 September 2021

Q1) Consider the correct statement about Comprehensive Economic Partnership Agreement: 1) Partnership agreements or cooperation agreements are more comprehensive than Free Trade

Agreements. 2) India has signed CEPAs with South Korea and Japan.

a) 1 only b) 2 only c) Both 1 and 2 d) Neither 1 nor 2

Page 80: DAILY Q & A SERIES

DAILY Q & A SERIES

For Enrolment Join Telegram Channel: SIMPLIFIED_GS

Ans : (c)

Recently, India and the United Arab Emirates (UAE) formally launched negotiations on the India-UAE Comprehensive Economic Partnership Agreement (CEPA).

Comprehensive Economic Partnership Agreement:

o It is a kind of free trade pact which covers negotiation on the trade in services and investment, and other areas of economic partnership. It may even consider negotiation on areas such as trade facilitation and customs cooperation, competition, and Intellectual Property Rights.

o Partnership agreements or cooperation agreements are more comprehensive than Free Trade Agreements.

o CEPA also looks into the regulatory aspect of trade and encompasses an agreement covering the regulatory issues.

o India has signed CEPAs with South Korea and Japan.

Q2 ) New Global Air Quality Guidelines Released by

a) UNEP

b) WWF

c) WHO

d) a & c

Ans: c)

Recently, the World Health Organisation (WHO) has released new Global Air Quality Guidelines (AQGs). Under these guidelines, WHO has further lowered the recommended levels of pollutants that can be considered safe for human health.

This is the first-ever update of WHO since 2005. The goal of the guideline is for all countries to achieve recommended air quality levels.

New Guidelines:

o The guidelines recommend new air quality levels to protect the health of populations, by reducing levels of key air pollutants, some of which also contribute to climate change.

o By striving to achieve these guideline levels, countries will be both protecting health as well as mitigating global climate change.

o WHO move sets the stage for eventual shifts in policy in the government towards evolving newer stricter standards.

Page 81: DAILY Q & A SERIES

DAILY Q & A SERIES

For Enrolment Join Telegram Channel: SIMPLIFIED_GS

o WHO’s new guidelines recommend air quality levels for 6 pollutants, where evidence has advanced the most on health effects from exposure.

6 classical pollutants include particulate matter (PM 2.5 and 10), ozone (O3), nitrogen dioxide (NO2) sulfur dioxide (SO2) and carbon monoxide (CO).

Q3 ) Consider the INCORRECT statement about Climate-Induced Migration and Modern Slavery report

:

1) International Institute for Environment and Development (IIED) and Anti-Slavery

International released a report named Climate-Induced Migration and Modern Slavery.

2) Anti-Slavery International is the world’s oldest international human rights organisation,

founded in 1839.

a) 1 only

b) 2 only

c) Both 1 and 2

d) Neither 1 nor 2

Ans : d)

Recently, International Institute for Environment and Development (IIED) and Anti-Slavery International released a report named Climate-Induced Migration and Modern Slavery.

IIED is a policy and action research organisation promoting sustainable development and linking local priorities to global challenges. It is based in London, UK.

Anti-Slavery International is the world’s oldest international human rights organisation, founded in 1839. It is the only British charity exclusively working to eliminate all forms of slavery.

Increasing Inequality:

o Climate change is devastating the planet, leading to intensifying global inequality as well as disputes over land, water and scarce resources.

Increased Migration:

o People are being driven to migrate within and across borders in search of resources and income.

As many as 55 million people were internally displaced within their countries due to extreme weather events in 2020.

Q4) Recently, CIM-Pitamber seen in the news, is related to

a) a high-yielding curcuminoid-rich variety of Turmeric. b) a high-yielding curcuminoid-rich variety of Onion.

Page 82: DAILY Q & A SERIES

DAILY Q & A SERIES

For Enrolment Join Telegram Channel: SIMPLIFIED_GS

c) a high-yielding curcuminoid-rich variety of Garlic. d) a high-yielding curcuminoid-rich variety of Cotton.

Ans : a)

Recently, a high-yielding curcuminoid-rich variety of turmeric named CIM-Pitamber and the NBRI’s (National Botanical Research Institute) Keshari variety have been introduced in Nabarangpur (One of the Aspirational Districts) of Odisha.

CIM-Pitamber:

o It is a high-yielding curcuminoid-rich variety of turmeric developed by Central Institute of Medicinal and Aromatic Plants (CIMAP).

High yielding varieties (HYV) of seeds are those seeds which produce huge quantities of crops particularly wheat and rice.

Regular supply of water, maximum use of fertilisers and use of pesticides in an accurate proportion is needed to use these seeds.

In this variety, curcuminoid content is 12.5% more than the existing variety.

Curcuminoid is a substance derived from turmeric which has anti-cancer properties, anti-inflammatory, anti-aging, anti-diabetic and has several medicinal properties.

Q5) National Single Window System for Investors and Businesses, recently seen in news, is an

initiative of:

a) Union Ministry of Commerce & Industry

b) Union Ministry of Finance

c) Union Ministry of Home Affairs

d) None of the above

Ans: a)

National Single Window System for Investors and Businesses was launched by Union Minister for Commerce & Industry Piyush Goyal.

This single window portal will become a one-stop-shop for investors for approvals & clearances. The portal as of today hosts approvals across 18 Central Departments & 9 States, another 14 Central

depts & 5 states will be added by Dec’21.

Page 83: DAILY Q & A SERIES

DAILY Q & A SERIES

For Enrolment Join Telegram Channel: SIMPLIFIED_GS

This would bring Transparency, Accountability & Responsiveness in the ecosystem and all information will.be available on a single dashboard. An applicant Dashboard would be there to apply, track & respond to queries.

Services include Know Your Approval (KYA), Common Registration & State registration Form, Document repository & E-Communication. Q6) Consider the INCORRECT statement about Solar DC Cooking System:

1) It is developed by DRDO.

2) It is a Solar Energy based Cooking System which consists of solar PV panel, charge controller, battery bank and cooking oven.

a) 1 only

b) 2 only

c) 1 and 2

d) Neither 1 nor 2

Ans : a)

CSIR-CMERI, handed over the CSIR-CMERI developed Solar DC Cooking System to the Asansol Braille Academy, West Bengal.

The CSIR-CMERI developed Solar DC Cooking System is a Solar Energy based Cooking System which consists of solar PV panel, charge controller, battery bank and cooking oven.

The technology provides a Clean Cooking Environment, Invertor-Less Direct Operation, Fast and Uniform Heating and a potential to save 1 ton Carbon Dioxide emissions year/household.

It has 20-25% better efficiency and more Economical in comparison with Conventional Solar based Cooking Systems which loses efficiency owing to AC-DC conversion.

This System will help in substantial curbing of Carbon Dioxide emissions; as even LPG usage emits Carbon Dioxide.

The Central Mechanical Engineering Research Institute (also known as CSIR-CMERI Durgapur or CMERI Durgapur) is a public engineering research and development institution in Durgapur, West Bengal. It is a constituent laboratory of the Indian Council of Scientific and Industrial Research (CSIR).

Q7) Consider the correct statement about Monoclonal antibodies:

1) They are artificially created antibodies that aim to aid the body’s natural immune system.

2) They target a specific antigen — a protein from the pathogen that induces immune response.

a) 1 only

b) 2 only

c) Both 1 and 2

d) Neither 1 nor 2

Ans : c)

Page 84: DAILY Q & A SERIES

DAILY Q & A SERIES

For Enrolment Join Telegram Channel: SIMPLIFIED_GS

European Commission signs contract for the supply of a monoclonal antibody treatment.

18 Member States have signed up to the joint procurement for the purchase of up to 220,000 treatments.

What are Monoclonal antibodies?

They are artificially created antibodies that aim to aid the body’s natural immune system.

They target a specific antigen — a protein from the pathogen that induces immune response.

How are they created?

Monoclonal antibodies can be created in the lab by exposing white blood cells to a particular antigen.

To increase the quantity of antibodies produced, a single white blood cell is cloned, which in turn is used to create identical copies of the antibodies.

In the case of Covid-19, scientists usually work with the spike protein of the SARS-CoV-2 virus, which facilitates the entry of the virus into the host cell.

Q8) Lag B’Omer Festival, recently seen in the news, is celebrated by which of the following ethnic groups?

(a) Persians

(b) Jews

(c) Greeks

(d) Latins

Ans : b)

Lag B’Omer Festival is an annual Jewish festival observed during the Hebrew month of Iyar. It is celebrated on the 33rd day of the Omer, the 49-day period between Passover & Shavuot (festivals) and is the only day during this period when celebration is permitted. Jews schedule weddings on this day & young boys are brought here for the first hair cut.

Q9) Xylophis deepaki, recently seen in the news, is a species of?

(a) Snakes

(b) Algae

(c) Butterflies

(d) Frogs

Ans : a)

Xylophis deepaki is a tiny wood snake of just 20 cm length with iridescent scales with a broad off-

white collar. It is an endemic species of Tamil Nadu and was first found in a coconut plantation of the

drier regions and in lower altitudes around Agasthyamalai hills in Kanyakumari. It is named in honour of

Page 85: DAILY Q & A SERIES

DAILY Q & A SERIES

For Enrolment Join Telegram Channel: SIMPLIFIED_GS

Indian herpetologist Deepak Veerappan for his contribution in erecting a new subfamily Xylophiinae to

accommodate wood snakes.

Q10) India is a party to which of the following given international convention(s)?

1. UN Refugee Conventions, 1951

2. Universal Declaration of Human Rights, 1948

3. International Covenant on Civil and Political Rights, 1966

(a) 1 only

(b) 2 and 3 only

(c) 1 and 3 only

(d) 1, 2 and 3

Ans: b)

India is not a party to the UN Refugee Conventions, 1951 but is a party to the Universal Declaration of

Human Rights, 1948 and the International Covenant on Civil and Political Rights, 1966.

24 September 2021

Q1) Lake Tanganyika, which was seen in the news, is shared by how many countries?

a) 2 b) 3 c) 4 d) 5

Ans : (c)

Lake Tanganyika, located in Africa is shared by four countries namely Tanzania, the Democratic Republic of the Congo (DRC), Burundi, and Zambia. This is the second-oldest, second deepest and the longest freshwater lake in the world. The lake is recently in news, as “Save the Children” an independent organisation for children has stated that significant rise of Lake Tanganyika due to climate shocks has led to 84% internal migration in Burundi. In Burundi, most people live along the shores of this lake.

Q2 ) Consider the correct statement about Defence Industrial Corridor.:

1) Uttar Pradesh Defence Industrial Corridor was announced by the Prime Minister while inaugurating the UP Investors Summit in Lucknow in 2020.

2) Government has established another Defence Industrial Corridor in Tamil Nadu. a) 1 only b) 2 only c) Both 1 and 2 d) Neither 1 nor 2

Page 86: DAILY Q & A SERIES

DAILY Q & A SERIES

For Enrolment Join Telegram Channel: SIMPLIFIED_GS

Ans: b)

Recently, the Prime Minister visited the exhibition models of the Aligarh node of the upcoming Uttar Pradesh Defence Industrial Corridor.

It was announced by the Prime Minister while inaugurating the UP Investors Summit in Lucknow in 2018.

The government has established another Defence Industrial Corridor in Tamil Nadu.

Defence Industrial Corridor of UP:

o It is an aspirational project that intends to reduce foreign dependency of the Indian Aerospace & Defence Sector.

o It will have 6 nodes – Aligarh, Agra, Kanpur, Chitrakoot, Jhansi and Lucknow.

o The Uttar Pradesh Expressways Industrial Development Authority (UPEIDA) is the nodal agency to execute this project in conjunction with various other state agencies.

o It aims to bring up the state as one of the largest & advanced Defence manufacturing hubs and put it on the world map.

Tamil Nadu Defence Industrial Corridor

It comprises Chennai, Tiruchirappalli, Coimbatore, Salem and Hosur. It will create new defence production facilities and promote clusters with necessary testing and certification facilities, export facilitation centres, technology transfer facilitation, etc.

Page 87: DAILY Q & A SERIES

DAILY Q & A SERIES

For Enrolment Join Telegram Channel: SIMPLIFIED_GS

o The corridor was inaugurated in 2019.

Q3 ) ‘Last Ice Area’ (LIA) recently seen in the news, is located in the

a) Antarctica Region

b) Arctic Region

c) Himalaya Region

d) None of these

Ans : b)

Recently, the Arctic sea ice reached its minimum extent, coming in at 4.72 million square miles. It is the 12th lowest on record and the record minimum melting of the ice occurred in 2012.

September marks the end of the summer sea ice melt season and the Arctic sea ice minimum, which means when sea ice over the Northern Hemisphere ocean reaches its lowest extent of the year.

The ‘Last Ice Area’ (LIA), located in the Arctic’s Ice north of Greenland, has also started melting earlier than what the scientists had expected.

Q4) Which of the following state government recently started Air Gun Surrender Campaign?

a) Kerala

b) Tamil Nadu

c) Goa

d) Arunachal Pradesh

Ans : d)

Appealing people to not hunt animals and birds, Union Minister for Environment said that air gun surrender campaign will be launched across the country and retired forest workers, representatives of social organizations etc. will be roped in for the same.

He described the Air Gun Surrender Campaign of the Arunachal Government as an important initiative to save animals and birds from poaching and for their conservation

It is giving very positive results said that it will be started across the country and States will also be requested for this.

Air guns do not require any license. In such a situation, many people in our country hunt birds through air guns in every district and village. Due to this, the number of many birds is rapidly decreasing.

Union Minister for Environment also visited Itanagar Biological Park, the world's first Hoolok Gibbon Breeding Canter and reviewed the work being done to save this species.

Page 88: DAILY Q & A SERIES

DAILY Q & A SERIES

For Enrolment Join Telegram Channel: SIMPLIFIED_GS

Q5) Consider the INCORRECT statement about holistic rejuvenation of Ganga River and its basin :

1) There is only one ganga museum in India.

2) Jharkhand government has recently started ‘Organic farming authority of Jharkhand’ to promote organic farming in the state especially along the Ganga.

a) 1 only

b) 2 only

c) both 1 and 2

d) neither 1 nor 2

Ans: a)

The Union Minister for Jal Shakti chaired a meeting of the Empowered Task Force including several central ministries, departments and the state governments for ensuring better coordination and convergence among agencies and programs for holistic rejuvenation of Ganga River and its basin.

Among Central Ministries, the Ministry of Agriculture and Farmers Welfare informed that total area under organic farming and agro-forestry increased from 23,840 Ha to 1,03,780 Ha.

Out of this, land for Organic Farming in Uttarakhand is 50,840 Ha, Uttar Pradesh is 42,180 Ha, Bihar is 16,060 Ha & Jharkhand is 4,540 Ha. along the Ganga.

The Ministry of Tourism informed that the Ganga Museum in Haridwar is now operational while the one in Rishikesh is ready for inauguration and another Museum in Patna has been sanctioned.

The Ministry is also working on developing tourists circuits along the Ganga in Uttar Pradesh, Uttrakhand and Bihar.

Uttarakhand has started 20 big and 410 small outlets to promote organic products along Char Dham yatra while Jharkhand government has started ‘Organic farming authority of Jharkhand’ to promote organic farming in the state especially along the Ganga. Q6) “Transforming Food Systems for Rural Prosperity report” has been released by which

organization?

a) UNESCO b) FAO c) IFAD d) NABARD

Ans : c)

“Transforming Food Systems for Rural Prosperity” report has been released by International Fund for Agricultural Development (IFAD) which is an international financial institution and an UN specialised agency. The organization is headquartered at Rome, Italy. The report has laid emphasis on government investment in farming as well as in small and medium enterprises which support farm activities.

Q7) Consider the correct statement about Objectives of NASA’s VIPER Mission.

Page 89: DAILY Q & A SERIES

DAILY Q & A SERIES

For Enrolment Join Telegram Channel: SIMPLIFIED_GS

1) To explore the Moon’s South Pole region

2) Evaluate the concentration of water as well as other potential resources on its surface.

a) 1 only b) 2 only c) Both 1 and 2 d) Neither 1 nor 2

Ans : c)

NASA has chosen a landing site for the Volatiles Investigating Polar Exploration Rover (VIPER).

The site is on the western edge of Nobile crater, near the lunar south pole. The crater was named after Italian polar explorer Umberto Nobile.

About the mission:

VIPER is a mobile robot. It is the first resource mapping mission on any other celestial body. NASA’s Commercial Lunar Payload Services (CLPS) will be providing the launch vehicle and

lander for what’s going to be a 100-day mission.

Objectives of the mission:

1. To explore the Moon’s South Pole region. 2. Help create lunar resource maps. 3. Evaluate the concentration of water as well as other potential resources on its surface.

Q8) Consider the INCORRECT statement about Sign Language Day:

1) The day was first celebrated in 2018 as a part of the International Week of the Deaf. 2) September 23 was chosen because on this date the World Federation of the Deaf

(WFD) was established in 1951.

a) 1 only

b) 2 only

c) Both 1 and 2

d) Neither 1 nor 2

Ans : d)

Indian Sign Language Research and Training Centre (ISLRTC), an autonomous body within the Department of Empowerment of Persons with Disabilities (Divyangjan) under the Ministry of Social Justice and Empowerment, will be celebrating ‘Sign Language Day’ on September 23, 2021.

The theme for International Day of Sign Languages 2021 is “We Sign For Human Rights”. The day was first celebrated in 2018 as a part of the International Week of the Deaf.

Page 90: DAILY Q & A SERIES

DAILY Q & A SERIES

For Enrolment Join Telegram Channel: SIMPLIFIED_GS

September 23 was chosen because on this date the World Federation of the Deaf (WFD) was established in 1951.

The use of sign languages has been recognised and promoted by the Convention on the Rights of Persons with Disabilities (CRPD). That is sign languages are equal in status to spoken language and obligates states parties to facilitate the learning of sign language and promote the linguistic identity of the deaf community.

Q9) Odisha Forest department proposed to fit ‘Radio Collars’ on which animals, to eliminate animal –

human conflict?

a) Tigers b) Elephants c) Deer d) Gharial

Ans : b)

The forest department of Odisha has proposed to fit radio collars on 7 Elephants of the state, to track

their movement and prevent them from entering the human settlements. It aims to eliminate animal –

human conflict.

Three elephants of Chandaka Wildlife Sanctuary and four elephants of the Similipal Tiger Reserve have

been selected for this purpose. The device is fitted with a GPS device and the location can be tracked real

time.

Q10) Andorra Country is a tiny, independent principality situated between

a) France and Germany

b) France and Spain

c) Portugal and Spain

d) Italy and Spain

Ans: b)

Andorra has joined the International Monetary Fund (IMF), to become its 190th member. Andorra is a

microstate situated between France and Spain. However, it is the largest microstate in Europe.

27 September 2021

Q1) The Brahma Committee, recently seen in news, is related to:

a) Make recommendations on land rights in Assam.

b) Cyber security

Page 91: DAILY Q & A SERIES

DAILY Q & A SERIES

For Enrolment Join Telegram Channel: SIMPLIFIED_GS

c) Cyclone and its impact on India’s coastline

d) None of the above

Ans : (a)

An eviction drive in Sipajhar in Assam’s Darrang district took a violent turn, leaving two dead and several injured, after clashes broke out between the police and protesters.

The eviction drive in Dholpur of Sipajhar, where primarily Bengali-speaking Muslims live, was aimed at removing “illegal encroachers” to free up government land for “landless indigenous communities”.

Portions of the land in Dholpur — as well as the larger Garukhuti area — have been a site of conflict for decades, with indigenous residents claiming their land has been usurped by migrants. What is the extent of encroachment of land in Assam?

Land has long been at the centre of ethnic contestations in Assam, with the common belief that the “indigenous” Assamese were losing its land to “migrants from Bangladesh”.

The Brahma Committee, formed by the previous Sarbananda Sonowal government to make recommendations on land rights, said in its interim report in 2017 that 63 lakh bighas of government land was under “illegal occupation”.

Q2 ) Recently seen in news, Group of Ministers (GoM) to rationalise GST rates is constituted under:

a) Basavaraj S. Bommai

b) Pinarayi Vijayan

c) MK Stalin

d) Arvind Kejriwal

Ans: a)

Setting the stage for an overhaul of the multiple tax rates under the Goods and Services Tax regime, the government has tasked a Group of Ministers (GoM), led by Karnataka Chief Minister Basavaraj S. Bommai, with proposing a rationalisation of tax rates and considering the merger of different tax slabs within two months.

Currently, the GST regime has five broad tax rate slabs of zero, 5%, 12%, 18% and 28%, with a cess levied over and above the 28% on some goods, and special rates for items like precious stones and diamonds.

Mr. Bommai’s seven-member group constituted by the Finance Ministry on Friday includes West Bengal Finance Minister Amit Mitra and Kerala Finance Minister K.N. Balagopal, as well as GST Council members from Goa, Bihar, Uttar Pradesh and Rajasthan.

The GoM shall review the current tax slab rates and recommend changes in the same as may be needed to garner required resources (and) review the current rate slab structure of GST, including special rates,

Page 92: DAILY Q & A SERIES

DAILY Q & A SERIES

For Enrolment Join Telegram Channel: SIMPLIFIED_GS

and recommend rationalisation measures, including merger of tax rate slabs, required for a simpler rate structure in GST.

Q3 ) Recently, an international team of researchers made the first direct detection of dark energy. The

experiment named is

a) ELEMENTS

b) AC001

c) XENON1T

d) DA2C001

Ans : c)

Recently, an international team of researchers made the first direct detection of dark energy. The experiment named XENON1T, is the world’s most sensitive dark matter experiment and was operated deep underground at the INFN Laboratori Nazionali del Gran Sasso in Italy.

Dark energy is the mysterious form of energy that makes up about 68% of the universe, and has intrigued physicists and astronomers for decades.

Q4) Consider the correct statement about Indira Gandhi National Centre for the Arts (IGNCA):

1) IGNCA was established in 1987 as an autonomous institution under the Ministry of Tourism.

2) IGNCA aims to complete mapping in 5,000 villages by the end of financial year 2021-2022.

a) 1 only b) 2 only c) Both 1 and 2 d) Neither 1 nor 2

Ans : b)

Recently, the National Mission on Cultural Mapping (NMCM) has been handed over to the Indira Gandhi National Centre for the Arts (IGNCA), which will start a trial run in 75 villages in October 2021.

IGNCA was established in 1987 as an autonomous institution under the Ministry of Culture, as a centre for research, academic pursuit and dissemination in the field of the arts.

IGNCA aims to complete mapping in 5,000 villages by the end of financial year 2021-2022.

The Ministry of Culture had approved the NMCM in 2017 to build a comprehensive database of artists, art forms and other resources from organisations under the Ministry.

o It is aimed at addressing the necessities of preserving the threads of rich Indian Art and Cultural Heritage, converting vast and widespread cultural canvas of India into an objective Cultural Mapping while creating a strong “Cultural Vibrancy” throughout the nation.

Page 93: DAILY Q & A SERIES

DAILY Q & A SERIES

For Enrolment Join Telegram Channel: SIMPLIFIED_GS

o It encompasses data mapping, demography building, formalising the processes and bringing all cultural activities under one umbrella for better results.

o The work on creating a database for folk arts and mapping of heritage of villages would be carried out over five years (From 2017).

o Volunteers from the Nehru Yuva Kendra Sangathan, the National Service Scheme and students of sociology and social work would be deputed to collect such data from villages.

Q5) Considerb the correct statement:

1) The Chenani-Nashri Tunnel is India’s longest highway tunnel and Asia’s longest bi-directional highway tunnel.

2) Z-Morh Tunnel is an under construction tunnel, 20 km away from the Zojila Pass, on Srinagar-Kargil-Leh highway.

3) Zojila Tunnel will be Asia’s longest road tunnel, to be built at an altitude of 11,578 m above sea level.

a) 1 and 2

b) 2 and 3

c) 1 and 3

d) 1,2 and 3

Ans: d)

The Union Minister for Road Transport and Highways will lay the foundation stone of various National Highway (NH) Projects and will review and inspect Z-Morh and Zojila Tunnel in the Union Territory of Jammu & Kashmir.

Shyama Prasad Mukherjee Tunnel: The Chenani-Nashri Tunnel renamed as Shyama Prasad Mukherjee Tunnel.

o It is not only India’s longest highway tunnel (9 km long) but also Asia’s longest bi-directional highway tunnel.

o It is located in the lower Himalayan mountain range, between Udhampur and Ramban in Jammu & Kashmir.

Banihal Qazigund Tunnel: It is the 8.5 km road tunnel at elevation of 1,790 m in the Pir Panjal range in the union territory of Jammu and Kashmir connecting Banihal and Qazigund.

Jawahar Tunnel: It is also called Banihal Tunnel or Banihal Pass. The length of the tunnel is 2.85 km.

o It is situated between Banihāl and Qazigund on NH 1A that has been renumbered NH 44.

o The tunnel facilitates round-the-year road connectivity between Srinagar and Jammu.

Nandni Tunnels: These tunnels are the series of four highway tunnels built under Nandni wildlife sanctuary on Jammu-Srinagar National Highway in Udhampur district.

Page 94: DAILY Q & A SERIES

DAILY Q & A SERIES

For Enrolment Join Telegram Channel: SIMPLIFIED_GS

o At a combined length of 1.4 km, the tunnels have reduced the distance and travel time between Jammu and Srinagar.

Pir Panjal Railway Tunnel: It is India’s longest transportation railway tunnel measuring 11.2 km long.

o The tunnel link, which is the only broad gauge mountain railway in India, stretches through the Pir Panjal mountain range between Quazigund and Baramulla.

o The tunnel section is a part of the 202km Udhampur – Srinagar – Baramulla rail link project undertaken by the Northern Railways.

Z-Morh Tunnel: It is an under construction tunnel, 20 km away from the Zojila Pass, on Srinagar-Kargil-Leh highway.

o The 6.5 km long tunnel will connect Gagangir directly to Sonamarg in Kashmir.

Zojila Tunnel: It is an under construction tunnel located to the north-east of Srinagar, on the Srinagar- Leh section of the NH 1.

o The 14.2 km road tunnel between Baltal and Minamarg.

o The Zojila Tunnel will be Asia’s longest road tunnel, to be built at an altitude of 11,578 m above sea level.

o It will ensure safer, all weather year round connectivity between Leh, Kargil and Srinagar.

Nilgrar Tunnels:

o The Nilgrar-I is a twin tube tunnel of 433 m length each.

o The Nilgrar twin tunnel-II is of 1.95 Km length each.

o Nilgrar-I and Nilgrar-II tunnels are part of the 18 Km long approach road to Zojila west portal.

Zojila tunnel will provide connectivity to Ladakh region Kargil, Drass and Leh.

o Chattergala Tunnel: It is an under construction road tunnel in Jammu and Kashmir.

The tunnel will be 6.8 km long and will connect Kathua and Doda districts of Jammu and Kashmir via basohli-bani through Chattergala.

Q6) Correct statement to the women in Indian judiciary :

1. Women constituted only about 30% of the subordinate judiciary.

2. Of the 1.7 million advocates, only 15% are women.

Page 95: DAILY Q & A SERIES

DAILY Q & A SERIES

For Enrolment Join Telegram Channel: SIMPLIFIED_GS

a) 1 only

b) 2 only

c) Both 1 and 2

d) Neither 1 nor 2

Ans : c)

Chief Justice of India N.V. Ramana backed 50% representation for women in judiciary.

“It is your right. It is not a matter of charity... Enough of this thousands of years of suppression,” Chief Justice Ramana said. The CJI paraphrased Karl Marx to say, “Women of the world, unite! You have nothing to lose but your chains.”

The CJI said women constituted only about 30% of the subordinate judiciary. In High Courts, women judges constitute 11.5%. Here in the Supreme Court, we currently have four women Justices out of the sitting 33. That makes it just 12%.

Of the 1.7 million advocates, only 15% are women.

Only 2% of the elected representatives in the State Bar Councils are women. There is no woman member in the Bar Council of India. This needs urgent correction,” Chief Justice Ramana said.

Q7) Which organization has released the report titled “Climate Indicators and Sustainable

Development: Demonstrating the Interconnections”?

a) UNFCCC b) UNICEF c) NITI Aayog d) World Meteorological Organization (WMO)

Ans : d)

The World Meteorological Organization (WMO) has released a report titled “Climate Indicators and Sustainable Development: Demonstrating the Interconnections”. The report stresses the need for immediate action and international collaboration for achieving the Sustainable Development Goals (SDGs) and to curtail global warming to less than 2 °C. WMO is a UN specialized agency, which is headquartered at Geneva, Switzerland.

Q8) Which tech initiative of India has bagged CIPS Excellence in Procurement Awards 2021, under the

“Best Use of Digital Technology” category?

a) SDG Dashboard b) GeM

Page 96: DAILY Q & A SERIES

DAILY Q & A SERIES

For Enrolment Join Telegram Channel: SIMPLIFIED_GS

c) Champions Portal d) GST Portal

Ans : b)

The Government of India’s Government e Marketplace (GeM) initiative has been adjudged as the winner of CIPS Excellence in Procurement Awards 2021, under the “Best Use of Digital Technology” category. The award ceremony was held in London, UK. GeM is a dedicated online marketplace for procurement of goods and services by Govt. Government Organisations, Departments and PSUs.

Q9) Consumer Price Index, consider the correct statements:

1. It monitors retail prices at a certain level for a particular commodity at all-India levels.

2. It gives an idea of the cost of the standard of living and helps understand the purchasing power of the nation’s currency.

3. It is calculated by multiplying 100 to the fraction of the cost price of the current period and the base period.

(a) 1 only

(b) 1 and 2 only

(c) 2 and 3 only

(d) 1, 2 and 3

Ans : d)

Recently, retail inflation has hit a six-month high of 6.3 per cent in May, due to persistent rise in fuel

and edible oil prices.

The Consumer Price Index (CPI) monitors retail prices at a certain level for a particular commodity;

price movement of goods and services at rural, urban and all-India levels.

It gives an idea of the cost of the standard of living. Generally, CPI is used as a macroeconomic indicator

of inflation, as a tool by the central bank and government for inflation targeting and for inspecting price

stability, and as a deflator in the national accounts. CPI formula: (Price of basket in current period/Price

of basket in base period) x 100.

Q10) Recently,Dahla Dam was in the news, it is located in which of the following countries?

(a) Pakistan

(b) Tajikistan

(c) Afghanistan

(d) Turkmenistan

Ans; c)

The Dahla Dam, also known as Arghandab Dam is located in the Shah Wali Kot District of Kandahar Province in Afghanistan. It was constructed in 1952 by the USA on the Arghandab River.

28 September 2021

Page 97: DAILY Q & A SERIES

DAILY Q & A SERIES

For Enrolment Join Telegram Channel: SIMPLIFIED_GS

Q1) Consider the correct statement : 1) Recently, the supreme court ruled that government aid to an institution is a matter of

Fundamental right and it is not a matter of policy.

2) Article 28 of the Constitution of India (dealing with rights of minorities to establish and administer educational institutions) is subject to its own restrictions being reasonable.

a) 1 only b) 2 only c) Both 1 and 2 d) Neither 1 nor 2

Ans : (d)

Recently, the Supreme Court (SC) ruled that government aid to an institution is a matter of policy and it is not a fundamental right.

Article 30 of the Constitution of India (dealing with rights of minorities to establish and administer educational institutions) is subject to its own restrictions being reasonable.

Aid not a Fundamental Right:

o The right of an institution, whether run by a majority or minority community, to get government aid is not a fundamental right. Both have to equally follow the rules and conditions of the aid.

Reason:

o Government aid is a policy decision. It depends on various factors including the interests of the institution itself and the ability of the government to understand the exercise.

o Financial constraints and deficiencies are the factors which are considered relevant in taking any decision while giving aid, including both the decision to grant aid and the manner of disbursement of an aid.

Withdrawal of Aid:

o If the government made a policy call to withdraw aid, an institution cannot question the decision as a matter of right.

o If an institution does not want to accept and comply with the conditions accompanying such aid, it is well open to it to decline the grant and move in its own way. On the contrary, an institution can never be allowed to say that the grant of aid should be on its own terms.

Q2 ) Consider the INCORRECT statement about Election Commission of India:

1) ECI is not empowered to de-register parties on the grounds of violating the Constitution or

breaching the undertaking given to it at the time of registration.

2) ECI has the power to register parties under the Representation of the People Act,(RPA) 1951.

a) 1 only

Page 98: DAILY Q & A SERIES

DAILY Q & A SERIES

For Enrolment Join Telegram Channel: SIMPLIFIED_GS

b) 2 only

c) Both 1 and 2

d) Neither 1 nor 2

Ans: d)

Recently, the Election Commission of India (ECI) has notified the updated list of registered political parties, seeking deregistration of parties which do not contest elections, also it raised concerns over misuse of income tax exemption law.

Page 99: DAILY Q & A SERIES

DAILY Q & A SERIES

For Enrolment Join Telegram Channel: SIMPLIFIED_GS

o There are over two thousands registered unrecognised parties, and the EC has sought the power to deregister (Cancel registration) inactive parties over fears of such entities misusing income tax exemption law.

o ECI also pushes for a wide range of electoral reforms to usher in digitisation, removing duplication of voters and enabling remote voting for Non-Resident Indians (NRIs), even migrant workers within the country.

Power of Deregistration:

o The ECI is not empowered to de-register parties on the grounds of violating the Constitution or breaching the undertaking given to it at the time of registration.

o ECI has the power to register parties under the Representation of the People Act,(RPA) 1951, but it does not have the power to deregister parties that are inactive.

o A party can only be de-registered if its registration was obtained by fraud; if it is declared illegal by the Central Government; or if a party amends its internal Constitution and notifies the ECI that it can no longer abide by the Indian Constitution.

Related Concern:

o It is suspected that several unrecognised parties may be engaged in money laundering, given the income tax exemptions they enjoy.

The Association of Democratic reforms, a watchdog NGO on electoral issues, has reported that the number of registered unrecognised political parties has increased twofold in the last 10 years.

o Section 13A of the Income Tax Act, 1961 has given 100% exemption to political parties on its income from house property, income from other sources, capital gains and voluntary contributions received from any person however, subject to certain conditions.

Q3 ) Consider the correct statement about Pradhan Mantri Formalisation of Micro food processing

Enterprises (PMFME) Scheme:

1. It is a central sector scheme.

2. It aims to enhance the competitiveness and promote formalization of the unorganized micro—food processing enterprises in India.

a) 1 only

b) 2 only

c) both 1 and 2

d) neither 1 nor 2

Page 100: DAILY Q & A SERIES

DAILY Q & A SERIES

For Enrolment Join Telegram Channel: SIMPLIFIED_GS

Ans : b)

The Ministry of Food Processing Industries in association with the Ministry of Housing and Urban Affairs, under the Pradhan Mantri Formalisation of Micro food processing Enterprises (PMFME) Scheme, virtually launched the Seed Capital Module.

It was launched on Deendayal Antyodaya Yojana-National Urban Livelihoods Mission (DAY-NULM) MIS Portal for seed capital assistance to members of urban Self Help Groups working in the food processing sector in India.

The seed capital portal can be accessed at https://nulm.gov.in/Auth/Login.aspx for Self Help Groups to avail the seed capital assistance of Rs.40,000 per SHG member under the PMFME Scheme. About the PMFME Scheme

Launched under the Aatmanirbhar Bharat Abhiyan, the Pradhan Mantri Formalisation of Micro food processing Enterprises (PMFME) Scheme is a centrally sponsored scheme that aims to enhance the competitiveness and promote formalization of the unorganized micro—food processing enterprises in India.

The scheme aims to empower the self-help groups by: o Seed capital support of Rs.40,000 per SHG member; o Credit linked subsidy for capital investment up to 35% with a ceiling of Rs. 10 lakh; o Credit linked grantsupport up to 35% for establishing common infrastructure; o Handholding support for DPR preparation and; o Capacity building and training support.

Q4) National Institute of Biotic Stress Management, recently seen in news, is located in:

a) Raipur

b) Chennai

c) Surat

d) Delhi

Ans : a)

To create mass awareness for adoption of climate resilient technologies, Prime Minister Shri Narendra Modi will dedicate 35 crop varieties with special traits to the Nation on 28th September During the programme.

The crop varieties with special traits have been developed by the Indian Council of Agricultural Research (ICAR) to address the twin challenges of climate change and malnutrition.

Thirty-five such crop varieties with special traits like climate resilience and higher nutrient content have been developed in the year 2021.

These include a drought tolerant variety of chickpea, wilt and sterility mosaic resistant pigeonpea, early maturing variety of soybean, disease resistant varieties of rice and biofortified varieties of wheat, pearl millet, maize and chickpea, quinoa, buckwheat, winged bean and faba bean.

Page 101: DAILY Q & A SERIES

DAILY Q & A SERIES

For Enrolment Join Telegram Channel: SIMPLIFIED_GS

About National Institute of Biotic Stress Management The Prime Minister will also dedicate to the nation the newly constructed campus of National Institute

of Biotic Stress Management Raipur. The National Institute of Biotic Stress Management at Raipur has been established to take up the basic

and strategic research in biotic stresses, develop human resources and provide policy support. The institute has started PG courses from the academic session 2020-21.

About Green Campus Awards On the occasion, the Prime Minister will distribute Green Campus Award to Agricultural Universities. The Green Campus Awards has been initiated to motivate the State and Central Agricultural Universities

to develop or adopt such practices that will render their campuses more green and clean, and motivate students to get involved in ‘Swachh Bharat Mission’, ‘Waste to Wealth Mission’ and community connect as per the National Education Policy-2020.

Q5) With reference to the foreign trade, consider the correct statements:

1. The Centre had earlier extended the foreign trade policy (FTP) 2015-20 until September 30 this year

due to the COVID-19 crisis.

2. The FTP provides guidelines for enhancing exports to push economic growth and create jobs.

a) 1 only

b) 2 only

c) Both 1 and 2

d) Neither 1 nor 2

Ans: c)

Commerce and Industry Minister Piyush Goyal said the existing foreign trade policy (FTP) will be extended till March 31, 2022.

The Centre had earlier extended the FTP 2015-20 until September 30 this year due to the COVID-19 crisis. The FTP provides guidelines for enhancing exports to push economic growth and create jobs.

Goyal also inaugurated a trade facilitation portal that was developed by Federation of Indian Export Organizations (FIEO) to help exporters deal with challenges due to acute shortage of containers. The portal will bring exporters and logistics service providers (LSPs) on a single platform. Q6) Recently, Meghalaya Enterprise Architecture Project (MeghEA) was launched. Correct the following statement:

1) Aims to improve service delivery and governance for the people using the power of Digital technologies.

Page 102: DAILY Q & A SERIES

DAILY Q & A SERIES

For Enrolment Join Telegram Channel: SIMPLIFIED_GS

2) The initiative is in line with the vision of Digital India.

a) 1 only

b) 2 only

c) Both 1 and 2

d) Neither 1 nor 2

Ans : c)

Recently, Meghalaya Enterprise Architecture Project (MeghEA) was launched.

The project aims to improve service delivery and governance for the people using the power of Digital technologies.

Enterprise Architecture (EA) is the process by which organizations standardize and organize IT (Information Technology) infrastructure to align with business goals.

o The initiative is spread across 6 pillars i.e. Governance, Human Resources, Entrepreneurship, Primary Sector, Infrastructure and Environment, and envision to make Meghalaya a high income state by 2030.

o The initiative is in line with the vision of Digital India that focuses on transforming India into a digitally empowered society and knowledge economy.

o With the notification of India Enterprise Architecture (IndEA) in 2018, design and implementation of integrated, nationally portable and scalable digital platforms have gained a new momentum.

Q7) Consider the INCORRECT statement about United Nations General Assembly:

1) Prime Minister of India (PM) addressed the 76th United Nations General Assembly.

2) Theme for this year is “Building Resilience through hope to recover from Covid-19, rebuild sustainably, respond to the needs of the planet, respect the rights of people, and revitalise the United Nations".

a) 1 only

b) 2 only

c) Both 1 and 2

d) Neither 1 nor 2

Ans : d)

Recently, the Prime Minister of India (PM) addressed the 76th United Nations General Assembly (UNGA).

The UNGA’s theme for this year is “Building Resilience through hope to recover from Covid-19, rebuild sustainably, respond to the needs of the planet, respect the rights of people, and revitalise the United Nations".

The PM spoke about a range of topics from the Covid-19 pandemic, threat of terrorism, India's actions to combat climate change and the need to protect the freedom of navigation in the oceans.

Q8) Consider the correct statement about Agni V Missiles:

Page 103: DAILY Q & A SERIES

DAILY Q & A SERIES

For Enrolment Join Telegram Channel: SIMPLIFIED_GS

1) Agni-V is the most advanced surface-to-air indigenously built ballistic missile.

2) Agni-V is a fire and forget missile.

3) It has been developed under the Integrated Guided Missile Development Programme.

a) 1 and 2

b) 2 and 3

c) 1 and 3

d) 1,2 and 3

Ans : b)

Recently, China has cited a United Nations Security Council (UNSC) resolution to question India’s missile programme amid reports of an upcoming test for the Agni-V intercontinental ballistic missile.

UNSC Resolution 1172 was issued after India's 1998 nuclear tests.

About Agni V Missiles:

o Agni-V is the most advanced surface-to-surface indigenously built ballistic missile.

o It is a three-stage, solid fuelled, 17-metre tall missile, and is capable of carrying a nuclear warhead of about 1.5 tonnes.

o Agni-V is a fire and forget missile, which once fired cannot be stopped, except by an interceptor missile.

o It has been developed under the Integrated Guided Missile Development Programme (IGMDP).

IGMDP was conceived by Dr. A.P.J. Abdul Kalam to enable India attain self-sufficiency in the field of missile technology. It was approved by the Government of India in 1983 and completed in March 2012.

The 5 missiles (P-A-T-N-A) developed under this program: Prithvi, Agni, Trishul, Nag, Akash.

o Agni Class of Missiles:

They are the mainstay of India’s nuclear launch capability.

Range:

Agni I: Range of 700-800 km.

Agni II: Range more than 2000 km.

Agni III: Range of more than 2,500 Km

Agni IV: Range is more than 3,500 km and can fire from a road mobile launcher.

Agni-V: The longest of the Agni series, an Inter-Continental Ballistic Missile (ICBM) with a range of over 5,000 km.

Page 104: DAILY Q & A SERIES

DAILY Q & A SERIES

For Enrolment Join Telegram Channel: SIMPLIFIED_GS

Agni-P (Prime): It is a canisterised missile with range capability between 1,000 and 2,000 km. It will replace the Agni I missile.

o The missile has been successfully tested five times and is in the process of induction into the Army.

o Very few countries, including the US, China, Russia, France and North Korea, have InterContinental Ballistic Missiles (ICBM).

ICBM is a land-based, nuclear-armed ballistic missile with a range of more than 5,600 km.

Q9) Which entity operates “Digital Sky” Platform in India?

a) NITI Aayog b) Reserve Bank of India c) Directorate General of Civil Aviation d) Ministry of Information Technology

Ans : c)

The “Digital Sky” Platform is operated by Directorate General of Civil Aviation (DCGA) in India.

Recently, the Union Ministry of Civil Aviation has come out with a airspace map, exclusively for drone

operations in India. This map can be accessed using the “Digital Sky” Platform.

As per the map, air space is bifurcated into green, red and yellow zones for drone operations.

Q10) Which Ministry launched the ‘Planetarium Innovation Challenge’ for Indian start-ups and

entrepreneurs?

a) Ministry of Science and Technology b) Ministry of Electronics and IT c) Ministry of Jal Shakti d) Ministry of MSME

Ans: b)

MyGov India, under the Ministry of Electronics and Information Technology (MeitY), has launched the

Planetarium Innovation Challenge for Indian start-ups and tech entrepreneurs.

> > The challenge aims to encourage tech firms and Indian start-ups to build an indigenous planetariums

system software using latest technologies like Augmented Reality (AR), Virtual Reality (VR) and Merged

Reality (MR). The applicants may be Start-ups, Indian Entities or Individuals or Teams.

29 September 2021

Q1) Consider the correct statement about Foreign Terrorist Organisation: 1) FTO are foreign organizations that are designated by the Secretary of State US.

Page 105: DAILY Q & A SERIES

DAILY Q & A SERIES

For Enrolment Join Telegram Channel: SIMPLIFIED_GS

2) Recently, the US Congressional report on terrorism stated that Pakistan is home to at least 12 groups designated as Foreign Terrorist Organisation. a) 1 only b) 2 only c) Both 1 and 2 d) Neither 1 nor 2

Ans : (c)

Recently, the US Congressional report on terrorism stated that Pakistan is home to at least 12 groups designated as Foreign Terrorist Organisation (FTO).

The report named Terrorist and Other Militant Groups in Pakistan, released by the bipartisan research wing of US Congress in the Quad summit 2021.

Earlier, in February 2021, the Financial Action Task Force (FATF) had decided to retain Pakistan on the greylist.

Foreign Terrorist Organisation

FTO are foreign organizations that are designated by the Secretary of State US. This plays a

critical role in fight against terrorism and is an effective means of curtailing support for terrorist

activities and pressuring groups to get out of the terrorism business.

Q2 ) Build Back Better World (B3W) Initiative is being seen as the US’ initiative to counter ?

a) International solar mission Alliance

b) China's Belt and Road Initiative

c) Taliban movement

d) b & c

Ans: b)

US officials are set to visit Latin America to look for possible projects for the Build Back Better World (B3W) Initiative. B3W is an international infrastructure investment initiative announced by the Group of Seven (G-7) richest democracies in June 2021.

The B3W initiative is being seen as the US’ initiative to counter China's Belt and Road Initiative (BRI).

B3W & its Guiding Principles:

o Goal: The Build Back Better Plan is a Covid-19 relief, future economic, and infrastructure package proposed by G-7 countries for developing and lower-income countries.

o Components of B3W: Through B3W, the G7 and other like-minded partners will coordinate in mobilizing private-sector capital in four areas of focus:

Climate,

Page 106: DAILY Q & A SERIES

DAILY Q & A SERIES

For Enrolment Join Telegram Channel: SIMPLIFIED_GS

Health and health security,

Digital technology,

Gender equity and equality.

Q3 ) Mekedatu Project associated with which river ?

a) Krishna river

b) Godawari river

c) Cauvery river

d) Brahmaputra river

Ans : c)

Recently, the Cauvery Water Management Authority (CWMA) directed Karnataka to immediately release the balance quantum of water to Tamil Nadu.

However, CWMA dropped discussion on the Mekedatu reservoir project following “strong protest” from Tamil Nadu, Kerala and Puducherry.

River Cauvery

It is known as ‘Ponni’ in Tamil, also known as Ganga of the south, and it is the fourth largest river of southern India.

It is a sacred river of southern India. It rises on Brahmagiri Hill of the Western Ghats in southwestern Karnataka state, flows in a southeasterly direction through the states of Karnataka and Tamil Nadu, and descends the Eastern Ghats in a series of great falls and drains into Bay of Bengal through Pondicherry.

o Some of its tributaries are Arkavathi, Hemavathi, Lakshmana Theertha, Shimsa, Kabini and Harangi.

Q4) Which of the following Organisation has launched the first-ever global strategy to defeat

meningitis - ‘Global Roadmap to Defeat Meningitis by 2030’.

Page 107: DAILY Q & A SERIES

DAILY Q & A SERIES

For Enrolment Join Telegram Channel: SIMPLIFIED_GS

a) UNEP

b) WHO

c) UNESCO

d) EU

Ans : b)

The World Health Organization (WHO) has launched the first-ever global strategy to defeat meningitis - ‘Global Roadmap to Defeat Meningitis by 2030’.

o Goals: The roadmap includes three visionary goals:

Eliminate epidemics of bacterial meningitis.

Reduce cases of vaccine-preventable bacterial meningitis by 50% and deaths by 70%.

Reduce disability and improve quality of life after meningitis of any cause.

o Aims:

Achievement of high immunisation coverage, development of new affordable vaccines and improved prevention strategies and outbreak response.

Speedy diagnosis and optimal treatment for patients.

Good data to guide prevention and control efforts.

Care and support for those affected, focusing on early recognition and improved access to care and support for after-effects.

Advocacy and engagement, to ensure high awareness of meningitis, accountability for national plans, and affirmation of the right to prevention, care and after-care services.

Meningitis:

o About: Meningitis is an inflammation (swelling) of the protective membranes covering the brain and spinal cord.

It is predominantly caused by bacterial and viral infection. However, injuries, cancer, certain drugs, and other types of infections also can cause meningitis.

o Symptoms: Severe headache that seems different from normal, Sudden high fever, Stiff neck, Confusion or difficulty concentrating, etc.

o Transmission: Most bacteria that cause meningitis such as meningococcus, pneumococcus and Haemophilus influenzae are carried in the human nose and throat.

They spread from person to person by respiratory droplets or throat secretions.

Page 108: DAILY Q & A SERIES

DAILY Q & A SERIES

For Enrolment Join Telegram Channel: SIMPLIFIED_GS

Group B streptococcus (bacteria) is often spread from mother to child around the time of birth.

Q5) Consider the INCORRECT statement about Integrated Guided-Missile Development Programme:

1) It was conceived by Dr. A.P.J. Abdul Kalam to enable India attain self-sufficiency in the field of

missile technology.

2) The 5 missiles (P-A-T-N-A) developed under this program .

a) 1 only

b) 2 only

c) Both 1 and 2

d) Neither 1 nor 2

Ans: d)

Recently, the Defence Research and Development Organisation (DRDO) tested a new version of the Akash Missile – ‘Akash Prime’ – from the Integrated Test Range (ITR), Chandipur, Odisha.

Earlier, DRDO launched the Akash-NG (New Generation) and the Man Portable Anti Tank Guided Missile (MPATGM).

Integrated Guided-Missile Development Programme

It was conceived by Dr. A.P.J. Abdul Kalam to enable India attain self-sufficiency in the field of missile technology. b

The 5 missiles (P-A-T-N-A) developed under this program are:

o Prithvi: Short range surface to surface ballistic missile.

o Agni: Ballistic missiles with different ranges, i.e. Agni (1,2,3,4,5)

o Trishul: Short range low level surface to air missile.

o Nag: 3rd generation anti-tank missile.

o Akash: Medium range surface to air missile.

Q6) Consider the correct statement about Aapada Mitra Programme

1) It is a centrally sponsored scheme.

2) NDMA (National Disaster Management Authority) is the implementing agency.

a) 1 only

b) 2 only

c) Both 1 and 2

d) Neither 1 nor 2

Page 109: DAILY Q & A SERIES

DAILY Q & A SERIES

For Enrolment Join Telegram Channel: SIMPLIFIED_GS

Ans : b)

Aapada Mitra Programme

Recently, the government has announced plans to have Disaster Management Volunteers (Aapda Mitras) in 350 districts of the country also released documents for Common Alerting Protocol (CAP)

CAP is a simple but general format for exchanging all-hazard emergency alerts and public warnings over all kinds of networks.

o It is a Central Sector Scheme that was launched in May 2016. NDMA (National Disaster Management Authority) is the implementing agency.

o It is a programme to identify suitable individuals in disaster-prone regions who can be trained to be first responders in times of disasters.

Aim:

o To provide the community volunteers with the skills that they would need to respond to their community’s immediate needs in the aftermath of a disaster thereby enabling them to undertake basic relief and rescue tasks during emergency situations such as floods, flash-floods and urban flooding.

Q7) Right to Information (RTI), consider the Incorrect statements:

1. The RTI Act came into force in June 2005.

2. The Whistleblowers Protection Act was passed by both Houses of Parliament, but never notified.

a) 1 only

b) 2 only

c) Both 1 and 2

d) Neither 1 nor 2

Ans : d)

Bihar has proved to be hostile terrain for Right to Information (RTI) activists, with as many as 20 having been killed since 2010. Six activists were killed in 2018 alone.

The RTI Act came into force in June 2005.

Page 110: DAILY Q & A SERIES

DAILY Q & A SERIES

For Enrolment Join Telegram Channel: SIMPLIFIED_GS

Sashidhar Mishra from Begusarai district was the first RTI activist killed in 2010 and just last week, Bipin Agrawal, 46, was killed, allegedly for exposing encroachment on government lands in East Champaran district.

Agrawal had filed at least 900 RTI applications seeking details on government land encroachment in the district.

In 2011, the Whistleblowers Protection Act was drafted. The Act was later renamed The Whistleblowers Protection Act, 2014 and was passed by both Houses of Parliament, but never notified.

Q8) Tai-Ahom and Matak’, which were seen in the news recently, are the communities demanding

Scheduled Tribe (ST) status in which state?

a) Andhra Pradesh b) West Bengal c) Assam d) Karnataka

Ans : c)

Tai-Ahom and Matak communities are demanding for being accorded the Scheduled Tribe (ST) status in the state of Assam. Assam Chief Minister Himanta Biswa Sarma held discussions with representatives of the two communities regarding their demand. All Assam Tribal Sangha (AATS), the apex tribal body of the State, is authorised by the government to recommend issuing ST certificates in the State. Besides the two, tea tribes, Koch-Rajbonshi, Moran and Chutia communities are also demanding ST status.

Q9) Consider the correct statements:

1. Vembanad is the longest lake in India, and the largest lake in the state of Tamil Naidu.

2. It is the second-largest Ramsar site in India only after the Sunderbans in West Bengal.

3. The Kumarakom Bird Sanctuary is located nearby to this lake.

(a) 1 and 2 only

(b) 2 and 3 only

(c) 1 and 3 only

(d) 1, 2 and 3

Ans; b)

On his first ‘Mann Ki Baat’ programe , the Prime Minister recognised Kottayam native NS Rajappan’s untiring effort in cleaning the Vembanad Lake. Option B is correct: Vembanad is the longest lake in India, and the largest lake in the state of Kerala. With an area of 2033 square kilometres, it is the second-largest Ramsar site in India only after the Sunderbans in West Bengal. It receives water from rivers like the Pamba and Periyar. The lake is a major source of fresh water for the state but also has brackish water areas. also known as Vembanad Kayal, Vembanad Kol, Punnamada Lake (in Kuttanad) and Kochi Lake (in Kochi). The lake witnessed Snake boat race known as Nehru Trophy Boat Race (locally known as Vallam Kali) in August. The Kumarakom Bird Sanctuary is located on the east coast of the lake.

Q10) Consider the not correctly matched pairs:

Reports — Releasing organisation

Page 111: DAILY Q & A SERIES

DAILY Q & A SERIES

For Enrolment Join Telegram Channel: SIMPLIFIED_GS

1. Global Risk Report — FATF

2. Global Financial Stability Report — World Bank

3. World Economic Outlook — World Economic Forum

(a) 1 and 2 only

(b) 2 and 3 only

(c) 1 and 3 only

(d) 1, 2 and 3

Ans: d)

Global Risk Report is released by the World Economic Forum (WEF) Global Financial Stability Report is released by the International Monetary Fund (IMF) World Economic Outlook is released by the International Monetary Fund (IMF).

30 September 2021

Q1) Consider the correct statement about recently launched PM-POSHAN scheme: 1) For providing one hot cooked meal in Government and Government-aided schools. 2) scheme will replace the existing Mid-day Meal Scheme. 3) The concept of TithiBhojan will be encouraged extensively.

a) 1 only b) 1 and 3 c) 1 and 2 d) 1,2 and 3

Ans : (d)

Recently, the Union Cabinet has approved the Prime Minister POSHAN scheme or PM-POSHAN for providing one hot cooked meal in Government and Government-aided schools.

The scheme will replace the existing national programme for mid-day meal in schools or Mid-day Meal Scheme.

It has been launched for an initial period of five years (2021-22 to 2025-26).

PM POSHAN:

Coverage:

The scheme will cover 11.8 crore students enrolled in classes 1 to 8 in over 11.2 lakh schools across the country.

Page 112: DAILY Q & A SERIES

DAILY Q & A SERIES

For Enrolment Join Telegram Channel: SIMPLIFIED_GS

Primary (1-5) and upper primary (6-8) schoolchildren are currently entitled to 100 grams and 150 grams of food grains per working day each, to ensure a minimum of 700 calories.

The scheme will be extended to students studying in pre-primary or Balvatikas running in government and government aided primary schools.

Balvatika is the pre-school that was started in government schools last year to include children aged younger than six years in the formal education system.

Nutritional Gardens:

The government will promote nutritional gardens in schools. The gardens are being provided to offer additional micro-nutrients to students.

Supplementary Nutrition:

The new scheme has a provision for supplementary nutrition for children in aspirational districts and those with high prevalence of anaemia.

It does away with the restriction on the part of the Centre to provide funds only for wheat, rice, pulses and vegetables.

Currently, if a state decides to add any component like milk or eggs to the menu, the Centre does not bear the additional cost. Now that restriction has been lifted.

Tithi Bhojan Concept:

The concept of TithiBhojan will be encouraged extensively.

TithiBhojan is a community participation programme in which people provide special food to children on special occasions/festivals.

Direct Benefit Transfer (DBT):

The central government will ensure Direct Benefit Transfer (DBT) from states to schools, which will use it to cover cooking costs.

Earlier money was allocated to the states, which then included their share of the money before sending it to a nodal midday meal scheme authority at district and tehsil levels.

This is to ensure no leakages at the level of district administration and other authorities.

Page 113: DAILY Q & A SERIES

DAILY Q & A SERIES

For Enrolment Join Telegram Channel: SIMPLIFIED_GS

Nutrition Expert:

A nutrition expert is to be appointed in each school whose responsibility is to ensure that health aspects such as Body Mass Index (BMI), weight and haemoglobin levels are addressed.

Social Audit of the Scheme:

A social audit of the scheme has also been mandated for each school in each state to study the implementation of the scheme, which was so far not being done by all states.

The Ministry of Education will also engage college and university students to monitor the scheme at a local level.

Fund Sharing:

The Centre will bear Rs. 54,061 crore of the total estimated cost of Rs 1.3 lakh crore, with the states paying Rs 31,733 crore (Rs 45,000 crore will be released by the Centre as subsidies for food grains).

Vocal for Local for Atmanirbhar Bharat:

Involvement of Farmers Producer Organizations (FPO) and Women Self Help Groups in implementation of the scheme will be encouraged.

Use of locally grown traditional food items for a fillip to local economic growth will be encouraged.

Q2 ) Consider the INCORRECT statement About Special Category Status for states:

1) Special category status is a classification given by the Centre to assist development of states that face geographical and socio-economic disadvantages.

2) This classification was done on the recommendations of the Fifth Finance Commission in 1969.

a) 1 only b) 2 only c) Both 1 and 2 d) Neither 1 nor 2

Ans: d)

Recently, the Bihar Government has asserted that it has not dropped the demand of special category status to Bihar.

About Special Category Status (SCS):

o Special category status is a classification given by the Centre to assist development of states that face geographical and socio-economic disadvantages.

Page 114: DAILY Q & A SERIES

DAILY Q & A SERIES

For Enrolment Join Telegram Channel: SIMPLIFIED_GS

o This classification was done on the recommendations of the Fifth Finance Commission in 1969.

o It was based on the Gadgil formula. The parameters for SCS were:

Hilly Terrain;

Low Population Density And/Or Sizeable Share of Tribal Population;

Strategic Location along Borders With Neighbouring Countries;

Economic and Infrastructure Backwardness; and

Nonviable Nature of State finances.

o SCS was first accorded in 1969 to Jammu and Kashmir, Assam and Nagaland. Since then eight more states have been included (Arunachal Pradesh, Himachal Pradesh, Manipur, Meghalaya, Mizoram, Sikkim, Tripura and Uttarakhand).

o There is no provision of SCS in the Constitution.

o Special Category Status for plan assistance was granted in the past by the National Development Council to the States that are characterized by a number of features necessitating special consideration.

Now, it is done by the central government.

o The 14th Finance Commission has done away with the 'special category status' for states, except for the Northeastern and three hill states.

Q3 ) Export Credit Guarantee Corporation (ECGC) is wholly owned by the

a) RBI

b) Ministry of Commerce and Industry

c) SEBI

d) None of these

Ans : b)

Recently, the Union Cabinet has approved capital infusion in the Export Credit Guarantee Corporation (ECGC) and its listing through an initial public offering.

The government will inject Rs 4,400 crore in the ECGC over a period of five years beginning 2021-22.

The Cabinet also approved continuation of the National Export Insurance Account (NEIA) scheme and infusion of Rs 1,650 crore Grant-in-Aid over five years.

About the ECGC:

o Establishment: The ECGC Ltd is wholly owned by the Ministry of Commerce and Industry.

Page 115: DAILY Q & A SERIES

DAILY Q & A SERIES

For Enrolment Join Telegram Channel: SIMPLIFIED_GS

The Government of India had initially set up the Export Risks Insurance Corporation in 1957.

After the introduction of insurance covers to banks during the period 1962-64, the name was changed to Export Credit & Guarantee Corporation Ltd in 1964.

It was changed to ECGC Ltd in August 2014.

o Objectives: ECGC was established to promote exports by providing credit insurance services to exporters against non-payment risks by the overseas buyers due to commercial and political reasons.

Q4) CIPET: Institute of Petrochemicals Technology is located in:

a) Jaipur

b) Chandigarh

C) Raipur

d) Mysuru

Ans : a)

Prime Minister Modi will inaugurate CIPET: Institute of Petrochemicals Technology, Jaipur.

Together with the Government of Rajasthan, the Government of India has established CIPET: Institute of Petrochemicals Technology, Jaipur.

It is self-sustainable and dedicatedly serves the needs of the petrochemical and allied industries. It will provide education to youth to become skilled technical professionals.

Q5) IndiaXports 2021 Portal, recently seen in news, is an initiative of:

a) Ministry of Finance

b) Ministry of Corporate Affairs

c) Ministry of Road Transport and Highways

d) Ministry of Micro, Small and Medium Enterprises

Ans: d)

Page 116: DAILY Q & A SERIES

DAILY Q & A SERIES

For Enrolment Join Telegram Channel: SIMPLIFIED_GS

Union Minister for MSMEs virtually inaugurated the India Export Initiative and IndiaXports 2021 Portal of India SME Forum in New Delhi.

IndiaXports aims to orient MSMEs free of cost, with the objective of focussing on the untapped export potential in existing tariff lines and supporting MSMEs in order to grow the number of exporting MSMEs and increase MSME exports by 50% in 2022 and contributing to the PM’s dream of the US $5 Trillion Economy.

This initiative features an Info Portal which serves as a knowledge base for exports by Indian MSMEs with the required information related to export potential for all the 456 tariff lines along with the potential markets as well as trends in exports, export procedures and lots more.

Apart from an export help desk, Instructor led orientation will also be provided to MSMEs through a series of sessions for specific sectors highlighting the opportunities in specific products in international markets.

The initiative targets 1 lakh+ MSMEs desirous of knowing more about exports and hand holding 30,000+ MSMEs to start exporting, doubling the base of active exporters. Q6) Consider the correct statement about Landsat 9:

1) It is an Earth monitoring satellite. 2) t is a joint mission of NASA and the ISRO.

a) 1 only b) 2 only c) Both 1 and 2 d) Neither 1 nor 2

Ans : a)

Landsat 9 was recently launched by NASA.

About Landsat 9:

It is an Earth monitoring satellite. It is a joint mission of NASA and the US Geological Survey (USGS). Together with Landsat 8, it will collect images of Earth’s surface. It takes 8 days to capture

the whole Earth. It is the most technologically advanced satellite of its generation. It can see more colour

shades with greater depths than the previous satellites, helping scientists capture more details about our ever-changing planet.

The instruments aboard Landsat 9 are the Operational Land Imager 2 (OLI-2) and the Thermal Infrared Sensor 2 (TIRS-2). They will measure different wavelengths of light reflected off the Earth’s surface.

About the Landsat series:

The first Landsat satellite was launched in 1972 and since then, Landsat satellites have collected images of our planet and helped understand how land usage has changed over the decades.

Page 117: DAILY Q & A SERIES

DAILY Q & A SERIES

For Enrolment Join Telegram Channel: SIMPLIFIED_GS

Q7) Which of the following initiatives resulted in the adoption of the founding principles of the Non-Aligned Movement (NAM)?

(a) Brasilia Declaration

(b) Bandung Conference

(c) Nairobi Declaration

(d) Vienna Convention

Ans : b)

The year 2021 marks the 66th anniversary of the landmark Bandung Conference that resulted in the adoption of the founding principles of the Non-Aligned Movement (NAM).

The basic concept for NAM originated in 1955 during discussions that took place at the Asia-Africa Bandung Conference held in Indonesia.

Q8) With reference to the Nuclear Fusion, Consider the following statements:

1. Heavy isotopes split apart to produce Energy.

2. It is the process that powers the Sun.

3. It takes place in a state of matter called plasma.

Which of the above statements is/are correct?

(a) 1 only

(b) 2 only

(c) 2 and 3 only

(d) 1, 2 and 3 only

Ans : c)

Recently, researchers at the Lawrence Livermore National Laboratory (which operates the National Ignition Facility in California, US) appeared to have demonstrated “fusion ignition” for the first time.

This breakthrough has brought the world closer to the dream of near-limitless clean energy through nuclear fusion.

They applied laser energy on fuel pellets to heat and pressurise them at conditions similar to that at the centre of our Sun.

This triggered the fusion reactions.

o These reactions released positively charged particles called alpha particles (helium), which in turn heated the surrounding plasma.

Page 118: DAILY Q & A SERIES

DAILY Q & A SERIES

For Enrolment Join Telegram Channel: SIMPLIFIED_GS

o The heated plasma also released alpha particles and a self-sustaining reaction called ignition took place.

o Ignition helps amplify the energy output from the nuclear fusion reaction and this could help provide clean energy for the future.

Significance of the Experiment: Reproducing the conditions at the centre of the Sun will allow studying:

o Plasma, the state of matter that has never been created in the lab before.

o Gain insights into quantum states of matter.

o Conditions closer and closer to the beginning of the Big Bang.

About Nuclear Fusion:

o Nuclear fusion is defined as the combining of several small nuclei into one large nucleus with the subsequent release of huge amounts of energy.

It is the opposite reaction of fission, where heavy isotopes are split apart.

o Harnessing fusion, the process that powers the Sun, could provide a limitless, clean energy source.

In the sun, the extreme pressure produced by its immense gravity creates the conditions for fusion to happen.

o Fusion reactions take place in a state of matter called plasma. Plasma is a hot, charged gas made of positive ions and free-moving electrons that has unique properties distinct from solids, liquids and gases.

Q9) Which of the following is/are applications of nanotechnology in the health sector?

1. Eye surgery

2. Detection of heart attack

3. Drug delivery

4. Removing toxins from the bloodstream

Select the correct answer using the code given below:

(a) 1 only

(b) 2 and 3 only

(c) 3 and 4 only

(d) 1, 2, 3 and 4

Ans : d)

A team of scientists has created a nanomicelle that can be used for effective drug delivery to treat various cancers including breast, colon and lung cancer.

Page 119: DAILY Q & A SERIES

DAILY Q & A SERIES

For Enrolment Join Telegram Channel: SIMPLIFIED_GS

Nanotechnology or nanotech is the technology that involves the manipulation of matter on atomic, molecular, and supramolecular scales. This includes particles of a scale of 1 to 100 nanometers.

Nanomicelles:

o Formation:

Nanomicelles are formed when amphiphilic molecules assemble themselves to create a globular structure that is only around 5 to 100nm in diameter.

Different agents are used to create nanomicelles, however, they are usually made through surfactant molecules that may be non-ionic, ionic, and cationic detergents. Some nanomicelles may also be developed from a mixture of lipids and detergents.

o Use in Drug Delivery:

They are amphiphilic, i.e. have a hydrophilic outer shell and a hydrophobic interior. This dual property makes them a perfect carrier for delivering drug molecules.

The hydrophilic shell makes the micelle water soluble that allows for intravenous delivery while the hydrophobic core carries a payload of drug for therapy.

Once injected intravenously, these nanomicelles can easily escape the circulation and enter the tumours where the blood vessels are found to be leaky. These leaky blood vessels are absent in the healthy organs.

Importance of Targeted Delivery:

o The goal for cancer therapy is destroying the cancer cells without harming healthy cells of the body.

o Chemotherapeutics approved for treatment of cancer are highly toxic with various side effects.

o Thus the need arises for effective targeted drug delivery.

Other Uses of Nanotechnology in Health Care:

o Nanotech detectors for heart attack.

o Nanochips to check plaque in arteries.

o Nanocarriers for eye surgery, chemotherapy etc.

o Diabetic pads for regulating blood sugar levels.

o Nanoparticles for drug delivery to the brain for therapeutic treatment of neurological disorders.

Page 120: DAILY Q & A SERIES

DAILY Q & A SERIES

For Enrolment Join Telegram Channel: SIMPLIFIED_GS

o Nanosponges are polymer nanoparticles coated with a red blood cell membrane, and can be used for absorbing toxins and removing them from the bloodstream.

o NanoFlares are used for detection of cancer cells in the bloodstream.

o Nanopores are used in making DNA sequencing more efficient.

Recent Use of Nanotechnology:

o Antiviral nano coating on face masks and Personal Protection Equipment (PPE) kits.

Risks of Nanotechnology:

o Since this field is still at its nascent stage, the likely risks are contentious.

o The regulatory authorities like the US Environmental Protection Agency and the Health and Consumer Protection Directorate of the European Commission have started assessing the potential risks posed by the nanoparticles.

o Nanotoxicology is the study of potential health risks of nanomaterials.

The human body can easily take up the nanomaterials as they are small in size. However, there is a need for detailed research on how it would behave inside an organism. The behaviour of nanoparticles based on their size, shape and surface reactivity must be thoroughly analysed before launching them into the market.

o Nanopollution is the generic term that is used to describe the waste generated by the nanodevices or nanomaterials during the manufacturing process.

Q10) Consider the following pairs:

Hydropower Projects Rivers

1. Tehri Dam Bhagirathi river

2. Tapovan Vishnugad Alaknanda river

3. Vishnugadh Pipalkoti Dhauliganga river

Which of the pairs given above is/are correctly matched?

(a) 1 only

(b) 1 and 2 only

(c) 2 and 3 only

(d) 1, 2 and 3

Ans: a)

The Tapovan Vishnugad Hydropower Plant is a 520 MW run-of-river hydroelectric

project being constructed on Dhauliganga River in Chamoli District of Uttarakhand,

India.

The Vishnugad Pipalkoti Hydro Electric Project (4 x 111 MW) is located on river

Alaknanda, a major tributary of river Ganga, in district Chamoli in the state of

Uttarakhand.

Page 121: DAILY Q & A SERIES

DAILY Q & A SERIES

For Enrolment Join Telegram Channel: SIMPLIFIED_GS